You are on page 1of 93

1. A self-referred patient is examined in physical therapy.

The physical therapist asks the patient


a variety of question in an attempt to rule out systemic involvement. Which of the following
questions would provide the most direct information on the presence of a brain tumor?

1. Have you had any unusual headaches or changes in your vision?


2. Can you think of any time during the past week when you may have fallen or being injured?
3. Have you ever had a sudden weight loss in the last three weeks without dieting?
4. Have you noticed any change in your bowel movements or flow of urination?

2. A physical therapist completes a normal muscle test on a patient that sustained a laceration
to the anterior surface of the forearm. When performing a test on the flexor pollicis brevis, the
therapist should direct the force to:

1. along the volar aspect of the proximal phalanx of the thumb


2. along the volar aspect of the distal phalanx of the thumb
3. along the dorsal aspect of the proximal phalanx of the thumb
4. along the dorsal aspect of the distal phalanx of the thumb

3. A physical therapist examines the cutaneous reflexes of a patient with suspected central
nervous system involvement. Which of the following is the most appropriate to utilize when
attempting to elicit Babinski reflex?

tuning fork
index finger
pointed end of a reflex hammer
cotton ball

4. A PT examines a 40 y/o old female referred to physical therapy after spraining her ankle
playing volleyball. During the examination, the patient exhibits extreme tenderness to palpation
over the sinus tarsi. What ligament is most often associated with tenderness of this area?
anterior talofibular
calcaneofibular
deltoid
posterior talofibular

5. A physical therapist assesses a patient’s lower extremity deep tendon reflexes using a reflex
hammer. Which of the following reflexes would provide the therapist with the most information
on the L3-L4 neurologic level?
1. patellar reflex
2. lateral hamstrings reflex
3. posterior tibial reflex
4. Achilles reflex

6. A physical therapist assesses a patient’s voice sounds as a part of a respiratory examination.


The therapist positions the stethoscope over the thorax and asks the patient to say “ninety
nine”. Which type of voice sound is assessed using this technique?

1. bronchophony
2. ego phony
3. pectoriloquy
4. pneumophony

7. A physical therapist observes the electrocardiogram of a patient during exercise. Which of the
following ECG changes would be considered during exercise?

1. increase amplitude of P wave


2. shortening of PR interval
3. ST segment depression of greater than 1mm
4. decrease in amplitude of T wave

8. A 32-year- old female is admitted to the hospital after sustaining extensive burns to her trunk
and right upper extremity. Which of the following burn classification would most likely require the
use of a graft?

1. superficial burn
2. superficial partial thickness burn
3. deep partial thickness burn
4. full-thickness burn

9. A 13-year-old boy is referred to physical therapy after being diagnosed with an injury to the
proximal tibial epiphysis. The boy was hit by a car and sustained a severe hyperextension force
applied to the left knee. What is the most serious complication of this injury?

1. lesion of the common peroneal nerve


2. tear of the posterior cruciate ligament
3. damage to the popliteal artery
4. patellar instability

10. A physical therapist examines the gait of a 32-year-old male with peripheral neuropathy. The
therapist observes that the patient’s right foot has a tendency to slap the ground during loading
response. The observation can be best explained by weakness of the:

1. iliopsoas
2. tibialis anterior
3. tibilais posterior
4. gastrocnemius

11. Physical therapist often utilize information obtained in the clinical setting for a variety of
educational purposes. Which of the following would represent an appropriate use of a medical
record without patient consent?

1. informally discussing a patient’s medical record with another patient


2. permitting an unauthorized person access to a patient’s medical record
3. utilization of the medical record as a part of quality assurance program
4. research anonymity is not preserved

12. A patient in work hardening program is required to lift packages weighing approximately 30
pounds overhead to a conveyor belt. The patient can complete the task , but is unable to
prevent excessive lumbar hyperextension while reaching for the conveyor belt. Which of the
following assumptions is most accurate?

1. additional weight should be added to the packages which will promote lumbar stability
2. the patient should continue lifting the 30 pound packages because he will gradually become
stronger
3. the task is too easy for the patient
4. the task is too difficult for the patient

13. A physical therapist instructs a patient who is unable to perform a standing transfer how to
utilize a sliding board. When using the sliding board to transfer from a wheelchair to a bed,
which wheelchair option is most desirable?

1. swing away detachable legrest


2. elevating legrest
3. full length, detachable armrest
4. adjustable height armrest

14. A patient diagnosed with T5 paraplegia is discharge from rehabilitation hospital following 16
weeks of therapy. Assuming a normal recovery, which of the following most accurately
describes the status of the patient’s bathroom transfers?

1. independent with the presence of the attendant


2. independent with adaptive devices and a sliding board
3. independent with bathroom adaptations
4. independent

15. A patient with patellar tracking dysfunction is examined in PT. Physical examination reveals
diminished VMO activity. The most appropriate to selectively train the VMO is:
1. quadriceps setting exercise and biofeedback
2. full arc terminal extension with manual resistance
3. SLR with leg weights
4. multiple angle isometrics

16. A physical therapist is treating a patient with a diagnosis of chronic arterial insufficiency.
Assuming the patient does not demonstrate pain at rest, which of the following treatment
techniques would be contraindicated for this patient?

1. ambulation with an assistive device


2. patient education regarding proper skin care
3. stationary cycling
4. ankle pumps with legs elevated

17. Which of the following goals is not realistic upon discharge from a phase 1 cardiac
rehabilitation program for a patient status post coronary artery bypass graft?

1. ambulate 100 feet on level surfaces


2. walk up and down a flight of stairs
3. locate and recognize changes in pulse rate
4. range of motion and exercises at 6 metabolic equivalents

18. A physical therapist attempts to obtain consent to participate in a formal aquatic exercise
program from a patient rehabilitating from multiple lower extremity lower extremity injuries
sustained in a motor vehicle accident. The therapist’s action is most representative of the ethical
principle termed:

1. autonomy
2. beneficence
3. nonmaleficence
4. justice

19. A physical therapist and physical therapist employed in an acute care hospital are
responsible for providing weekend therapy coverage. After examining the patient treatment list,
the therapists attempt to develop an action plan. Which of the following activities would be the
least appropriate for the physical therapist assistant?

1. instruct a patient in prosthetic donning and doffing


2. assist a patient with ambulation activities
3. examine a patient referred to physical therapy for instruction in a home exercise program
4 perform goniometric measurement on a patient two days status post anterior cruciate
ligament reconstruction
20. A patient that has been on extended bed rest is positioned on tilt table. After slightly
elevating the head of the tilt table, the patient begins to demonstrate signs of orthostatic
hypotension. The therapist’s most immediate response should be to:

1. reassure the patient that her response is not unusual


2. contact the director of the rehabilitation for assistance
3. document the incident in the patient’s chart
4. lower the tilt table

21. A physical therapist enters a remote storage area to retrieve a piece of equipment and
observes flames and smoke throughout the room. The most appropriate immediate action is:

1. attempt to extinguish the fire


2. remove the patients from the physical therapy area
3. announce the code for fire over the facility’s public address system
4. attempt to contain the fire to the storage room

22. A physical therapist conducts a home health visit for the patient status post transtibial
amputation that lives in a small two room efficiency. The therapist treats the patient for a small
wound on the anterior surface of the residual limb. After completing the treatment session, the
therapist would like to wash her hands. Which of the following action would be the most
inappropriate?

1. wash hands thorough in the kitchen sink


2. utilize liquid soap instead of bar soap
3. use paper towels to turn the faucets on and off
4. rub hands together under flowing water with soap for at least 10 seconds

23. A physical therapy department guidelines for electrical equipment care and service. Which
of the following does not meet acceptable equipment care and service standards?

1. AC power receptacles and plugs should be hospital grade quality


2. electrical equipment should be inspected every 24-36 months
3. a file of clinical and technical information for each piece of equipment should be established
4. documentation of inspection and repair activities should be available for each device

24. A patient with lower back injury rings a call bell and informs the physical therapist that the
hot pack is too intense. Assuming the patient has had the hot pack on for three minutes, the
most appropriate initial action is to:

1. check the patient’s skin


2. add additional towel layers
3. select another superficial heating agent
4. document the incident in the medical record

25. A physical therapist administers neuromuscular electrical stimulation to the quadriceps using
a bipolar electrode configuration. After observing the muscle contraction, the therapist decides
to modify the treatment set up in order to increase the depth of current penetration. The most
appropriate action is to:

1. utilize carbon-rubber electrodes


2. increase the size of the electrodes
3. utilize additional electrodes using bifurcated lead
4. increase the distance between the electrodes

25. A physical therapist applies silver sulfadiadiazine to the dorsum of the hand of the patient
with a deep partial-thickness burn. Which statement best describes the amount of the topical
agent that should be applied to the wound?

1. an amount provides the burn with a glistening appearance


2. an amount that makes it impossible to see through
3. an amount that is inversely proportional to the depth of the wound
4. an amount that is equal to the ounce for each square centimeter of the burn

26. A physical therapist provides preoperative instructions to a 21-year-old college student


scheduled for knee surgery. During the session, the patient expresses concern about his ability
to balance the demands of rehabilitation and his school work. The most appropriate therapist
response is:

1. ask the patient if he has considered taking a leave of absence


2. inform the patient that the rehabilitation must take priority over school work
3. remind the patient about the importance of rehabilitation
4. encourage the patient to pursue university resources to assist him with the transition
following surgery

27.
A physical therapist prepares an inservice on repetitive use injuries for a group of administrative
assistants. A part of the presentation, the therapist develops learning objectives. Which of the
following objectives would be considered in the cognitive domain?

1. list three potential consequences of an improperly designed work station


2. correctly adjust the level of a computer keyboard
3. devote five minutes in the morning and afternoon for stretching exercises
4. demonstrate proper posture when sitting at a desk

28. A physical therapist transports a patient with the brain injury to the physical therapy gym.
Each day after arriving in the gym, the patient asks the therapist, “Where am I?”. Recognizing:
1. You know where you are
2. You are in the same place, you were yesterday, at this time
3. You are in the physical therapy gym for your treatment session
4. You are in the hospital because of your injury

29. A physical therapy administers a submaximal exercise test using cycle ergometer. The test
consist of a warm up followed by three stages of increasing increments of work each lasting
three minutes and a cool down. During the second stage of exercise test the therapist records
the patient’s heart rate as 122bpm after two minutes and 134bpm after 3 minutes. Which of the
following would be the most appropriate therapist action?

1. administer a rating of perceived exertion scale


2. maintain the present work rate for an additional minute
3. progress to a specified rate for the third stage
4. discontinue the submaximal exercise test

30. A physical therapist reviews the medical record of a patient rehabilitating from knee surgery.
A recent entry in the medical record uses the term effusion. Which description most
appropriately defines the term?

1. increase volume of fluid


2. increase volume of fluid within the joint capsule
3. increase volume of fluid in the soft tissue external to the joint
4. increase volume of fluid in the joint capsule and the soft tissue external to the joint.

31. A physical therapist treats a patient with C5-C6 tetraplegia. During the treatment session the
patient’s spouse asks a questions regarding the patient’s ability to transfer independently
following rehabilitation. The most appropriate therapist response is to:

1. refer to spouse to the director of rehabilitation


2. refer to spouse to the patient’s primary physician
3. refer the spouse to the patient’s primary nurse
4. answer the spouse’s question

32. A patient diagnosed with peripheral vascular disease is examined in physical therapy. Which
of the following objective findings would result in an ambulation exercise program being
contraindicated?

1. decrease peripheral pulses


2. resting claudication
3. increased resting systolic blood pressure
4. decreased lower extremity strength
33. A patient recovering from serious hamstrings strain is examined isokinetically prior to
returning to track competition. Results of the examination reveal peak torque measurements of
125 ft. lbs with knee extension and 78 ft lbs. With knee flexion at 180 degrees per second on the
involved lower extremity. What conclusion can be made regarding the patient’s ability to return
to athletic competition?

1. the patient’s quadriceps and hamstrings strength are appropriate for a return to athletics
activities
2. the patient’s hamstrings strength demonstrates the need for continued rehabilitation
3. the patient’s quadriceps/hamstrings ratio is below acceptable levels for athletic activities.
4. Not enough information is given to make an accurate determination of the patient’s ability to
return to athletic competition

34. A patient recovering from serious hamstrings strain is examined isokinetically prior to
returning to track competition. Results of the examination reveal peak torque measurements of
125 ft. lbs with knee extension and 78 ft lbs. With knee flexion at 180 degrees per second on the
involved lower extremity. What conclusion can be made regarding the patient’s ability to return
to athletic competition?

1. the patient’s quadriceps and hamstrings strength are appropriate for a return to athletics
activities
2. the patient’s hamstrings strength demonstrates the need for continued rehabilitation
3. the patient’s quadriceps/hamstrings ratio is below acceptable levels for athletic activities.
4. Not enough information is given to make an accurate determination of the patient’s ability to
return to athletic competition

35. A physical therapist that recently returned form maternity leave reviews her daily patient
schedule. Her first patient is a patient is a 46-year-old male that is referred to physical therapy
for wound debridement. The patient’s medical record indicates that he is HIV positive. The most
appropriate therapist action is:

1. have support personnel complete the treatment under direct supervision


2. ask another therapist to treat a patient
3. treat the patient
4. contact the referring physician to discuss the treatment orders

36. A patient in a rehabilitation hospital returns to physical therapy after a meeting with his
physiatrist. The patient indicates that during the meeting the physiatrist discussed the effect of
his spinal cord injury on sexual function. Which of the following statements is typically not
accurate for a male patient with complete T7 paraplegia?

1. the patient will be able to achieve an erection


2. the patient will be able to ejaculate
3. the patient’s fertility will be diminished
4. the patient’s medications may affect his sexual function

37. A physical therapist reviews the medical record of a patient with Hodgkin’s disease. The
patient is a 45-year-old male who recently began chemotherapy treatment. What effect will
chemotherapy have on the patient’s ability to participate in a rehabilitating program?

1. the patient may be susceptible to infection


2. the patient may experience excessive fatigue
3. the patient may demonstrate cardiac anomalies
4. the patient may exhibit signs and symptoms of gastrointestinal distress.

38. A physical therapist receives a referral for a patient who is one-week status post CVA. When
observing the patient lying in bed, the therapist notes that the patient’s calf and foot are
edematous. The patient reports that the area is somewhat painful. The therapist should:

1. Discontinue the examination and hope the patient’s leg is better tomorrow.
2. consider ordering compression stockings for the patient
3. continue with the examination and disregard the patient’s condition
4. inform the physician of the situation and discontinue the examination

39. A physical therapist discusses a patient care plan with the physical therapist assistant in
preparation for treatment. The physical therapist indicates that the patient has ideomotor
apraxia. This condition is most consistent with:

1. difficulty performing sequenced motor acts


2. inability to carry out purposeful movement on command
3. failure to recognize familiar objects
4. inability to respond to stimuli presented contralateral to the side of a brain lesion

40. A patient rehabilitating from a CVA is referred to physical therapy. The medical records
indicate the CVA primarily involved the right hemisphere of the brain. Which of the following
objective findings would be the least likely when examining the patient?

1. diminished motor control of the left side of the body


2. impaired awareness of the right side of the body
3. impaired spatial ability
4. diminished awareness of disability

41. A patient with latissimus dorsi and trapezius weakness would have the most difficulty
performing which of the following activities?

1. four- point gait with lofstrand crutches


2. three- point gait with a straight cane
3. swing through gait with crutches
4. wheelchair propulsion

42. A physical therapist instructs a patient how to rise from a chair before beginning ambulation
activities with a walker. Which of the following instructions would be helpful to the patient?

1. place both hands on the walker and pull yourself to a standing position
2. push up on the chair with one hand and place the other hand on the edge of the walker for
balance
3. push up on the chair with both hands and reach for the walker once you are standing
4. push up on the chair with both hands and reach for the walker while rising

43. A patient rehabilitating from a fractured humerus has completed six weeks of physical
therapy and is ready to be discharged with a home exercise program. The patient is extremely
pleased with his progress in therapy and gives the physical therapist a check for $50.00 as a
token of his appreciation. The most appropriate therapist action is to:

1. accept the gift


2. accept the gift and donate to charity
3. accept the gift and donate it to the departments general expense fund
4. explain to the patient that you are not permitted to accept the gift

44. A physical therapist awaiting the arrival of her next patient observes another patient
ambulating independently in the parallel bars. The patients appears to lack the necessary
strength and coordination required to complete the activity independently. The therapist’s most
appropriate response would be:

1. inform the patient’s therapist of her observation


2. assist the patient back to a chair and contact the patients therapist
3. ask the patient if she is having difficulty or need any assistance
4. continue to observe the patient, but do not interfere

45. A physical therapist examines a 40-year old female referred to physical therapy after
spraining her ankle playing volleyball. During the examination, the patient exhibits extreme
tenderness to palpation over the sinus tarsi. What ligament is most often associated with
tenderness in this area?

1. anterior talofibular
2. calcaneofibular
3. deltoid
4. posterior talofibula
46. A patient diagnosed with an anterior cruciate ligament injury is examined in physical
therapy. During the examination, the patient asks the physical therapist why the physician order
x-rays after already diagnosing the ligament injury. The primary purpose for ordering the
radiographs would be to:

1. confirm the physician’s diagnosis


2. check the possible meniscal involvement
3. examine the patient’s skeletal maturity
4. rule out the possibility of a fracture

47. A physical therapist utilizes joint mobilization techniques for pain control and muscle
relaxation at the shoulder. If the therapist begins mobilizing the glenohumeral joint in the resting
position, the limb should be positioned in:

1. 55 deg of abduction, 30deg of horizontal adduction


2. 30deg of abduction, 10deg of horizontal adduction
3. 25 deg of abduction, 5 deg of horizontal abduction
4. 10 deg of abduction, 5 deg of horizontal abduction

48. A physical therapist transports a patient with the brain injury to the physical therapy gym.
Each day after arriving in the gym, the patient asks the therapist, “Where am I?”. Recognizing:

1. You know where you are


2. You are in the same place ,you were yesterday, at this time
3. You are in the physical therapy gym for your treatment session
4. You are in the hospital because of your injury

49. A physical therapist examines a patient with low back pain. Diagnostic imaging reveals the
patient has a mild disc protrusion L4-L5 level. Physical examination reveals a diminished lumbar
curve and the presence of a lateral shift. The most appropriate treatment intervention is:

1. instruct the patient in active extension exercises


2. instruct the patient in passive extension exercises
3. attempt to correct lateral shift
4. apply palliative modalities

50. Documentation from an orthopedic surgeon’s report indicates that a patient sustained
damage to several structures that provide anterolateral stability of the knee. Which of the
following structures would most likely be involved?

1. anterior cruciate ligament, posterior oblique ligament, iliotibial band


2. anterior cruciate ligament, medial collateral ligament, medial meniscus
3. anterior cruciate ligament, lateral collateral ligament, iliotibial band
4. posterior cruciate ligament, lateral collateral ligament, biceps femoris tendon.

1. While assessing the standing posture of a patient, the therapist notes that a spinous process
in the thoracic region is shifted laterally. The therapist estimates that T2 is the involved vertebra
because he or she notes that it is at the approximate level of the

A. inferior angle of the scapula


B. superior angle of the scapula
C. spine of the scapula
D. xiphoid process of the sternum

2. A patient comes to the therapist because she has noted a pronounced tuft of hair on the
center of her spinal column in the lumbar area. The therapist notes no loss in motor or sensory
function. This patient most likely has what form of spina bifida?

A. Meningocele
B. Meningomyelocele
C. Spina bifida occulta
D. Syringomyelocele

3. Which of the following is contraindicated to ultrasound at 1.5 watts/cm2 with a 1 MHz sound
head?

A. Over a recent fracture site.


B. Over non-cemented metal implant.
C. Over a recently surgically repaired tendon.
D. Over the quadriceps muscle belly.
4. The therapist is treating a 52-year-old woman after right total hip replacement. The patient
complains of being self-conscious about a limp. She carries a heavy briefcase to and from work
every day. The therapist notes a Trendelenburg gait during ambulation on level surfaces. What
advice can the therapist give the patient to minimize gait deviation?

A. Carry the briefcase in the right hand


B. Carry the briefcase in the left hand
C. The patient should not carry a briefcase at all.
D. It does not matter in which hand the briefcase is carried.

5. The therapist is evaluating a 32-year-old woman for complaints of right hip pain. The patient
has injured the strongest ligament of the hip. The therapist places the patient in the prone
position on the plinth and passively extends the involved hip. The therapist notes an abnormal
amount of increase in passive hip extension. Which of the following ligaments is damaged?
A. Ischiofemoral ligament
B. Iliofemoral ligament (Y ligament of Bigelow)
C. Pubofemoral ligament
D. Ligamentum teres

6. You are teaching a patient how to use a cane to assist with ambulation in CVA patient. The
cane is usually used:

Opposite the involved side


On the involved side
Only during the stance phase
As needed

7. On examination of a cross section of the spinal cord of a cadaver, the examiner notes
plaques. The finding is most characteristic of what condition?

A. Parkinson’s disease
B. Myasthenia gravis
C. Multiple sclerosis.
D. Dementia.

8. Which of the following is a contraindication to a 1-MHz ultrasound at 1.5 watts/cm2?

Over an injured tendon


Over an injured muscle belly
Over a non-cemented metal implant
Over an inflamed bursa

9. A therapist is mobilizing a patient’s right shoulder. The movement taking place at the joint
capsule is not completely to end range. It is a large amplitude movement from near the
beginning of available range to near the end of available range. What grade of mobilization
according to Maitland, is being performed?

A. Grade I
B. Grade II
C. Grade III
D. Grade IV

10. A physical therapist should place the knee in which of the following positions to palpate the
lateral collateral ligament (LCL)?

A. Knee at 60 deg of flexion and the hip externally rotated.


B. Knee at 20 deg of flexion and the hip at neutral.
C. Knee at 90 deg of flexion and hip externally rotated.
D. Knee at 0 deg and the hip at neutral.

11. The therapist is treating a patient with a T4 spinal cord injury when the patient suddenly
complains of a severe headache. The therapist also notes that the patient’s pupils are
constricted and that the patient is sweating profusely. Which of the following is the best course
of action for the therapist?

A. Try to find a probable source of noxious stimulus and position the patient supine with feet
elevated.
B. Try to find a probable source of noxious stimulus and position the patient with upper trunk
elevated and legs lowered.
C. Try to find a probable source of noxious stimulus and place the patient in a sidelying position.
D. Try to find a probable source of noxious stimulus and position the patient in prone position.

12. What lobe of the lungs is the therapist attempting to drain if the patient is in the following
position? Resting on the left side rolled 1⁄4 turn back, supported with pillows, and the foot of the
bed raised 12-16 inches.

A. Right middle lobe – lingular segment.


B. Left upper lobe – lingular segment
C. Right upper lobe – posterior segment
D. Left upper lobe – posterior segment.

13. A 25-year-old woman has been referred to a physical therapist by an orthopedist because of
low back pain. The therapist is performing an ultrasound at the L3 level of the posterior back
when the patient suddenly informs the therapist that she is looking forward to having her third
child. On further investigation, the therapist discovers that the patient is in the first trimester of
pregnancy. Which of the following is the best course of action for the therapist?

A. Change the settings of the ultrasound from continuous to pulsed.


B. Continue with the continuous setting because first trimester pregnancy is not a
contraindication.
C. Cease treatment, notify the patient’s orthopedic physician, and document the mistake.
D. Send the patient to the gynecologist for an immediate sonogram.

14. The therapist is ambulating a 42-year-old man who has just received an above knee
prosthesis for the left leg. The therapist notices pistoning of the prosthesis as the patient
ambulates. Which of the following is the most probable cause of this deviation?

A. The socket is too small


B. The socket is too large
C. The foot bumper is too soft.
D. The foot bumper is too hard.
15. A physical therapy technician calls the therapist immediately to the other side of the
outpatient clinic. The therapist discovers a 37-year-old female lying face down on the floor.
Which of the following sequence of events is most appropriate for this situation?

A. Have someone call 911, determine unresponsiveness, establish an airway, and assess
breathing (look/listen/feel).
B. Determine unresponsiveness, have someone call 911, establish an airway, and assess
breathing(look/listen/feel).
C. Have someone call 911, determine unresponsiveness, assess breathing (look/listen/feel).
D. Determine unresponsiveness, have someone call 911, assess breathing (look/listen/feel),
establish an airway.

16. The therapist observes a patient with the latter stages of Parkinson’s disease during
ambulation. Which of the following characteristics is the therapist most likely observing?

A. Shuffling gait
B. Increased step width
C. Difficulty initiating the first steps
D. AandC

17. The therapist is performing an orthopedic test that involves: (1) placing the patient in a
sidelying position, (2) placing the superior lower extremity in hip extension and hip abduction,
(3) placing the knee of the superior lower extremity in 90o of flexion, and (4) allowing the
superior lower extremity to drop into adduction. Failure of the superior lower extremity to drop
indicates a tight:

A. iliopsoas
B. rectus femoris
C. iliotibial band
D. hamstring

18. During the evaluation of an infant, the therapist observes that with passive flexion of the
head the infant actively flexes the arms and actively extends the legs. Which of the following
reflexes is being observed?

A. Protective extension
B. Optical righting
C. Symmetrical tonic neck
D. Labyrinthine head righting

19. A diabetic patient is exercising vigorously in an outpatient clinic. The patient informs the
therapist that he or she received insulin immediately before the exercise session. If the patient
goes into a hypoglycemic coma, which of the following is not a likely sign?
A. Pallor
B. Shallow respiration
C. Bounding pulse
D. Dry skin

20. A physician instructs the therapist to educate a patient about the risk factors of
atherosclerosis. Which of the following is the most inappropriate list?

A. Diabetes, male gender, and excessive alcohol


B. Genetic predisposition, smoking, and sedentary lifestyle
C. Stress and inadequate exercise
D. Obesity, smoking and hypotension

21. A physician is preparing a patient for an upcoming procedure. The physician explains that
the procedure will provide a detailed image that appears to be a slice of the brain. This image is
obtained with a highly concentrated x-ray beam. What procedure is the patient scheduled to
undergo?

A. Angiogram
B. Magnetic resonance imaging (MRI)
C. Positron emission tomography (PET)
D. Computed tomography (CT)

22. A patient at an outpatient facility experiences the onset of a grand mal seizure. Which of the
following is the most appropriate course of action by the therapist?

A. Assist patient to a lying position, move away close furniture, loosen tight clothes, and prop
the patient’s mouth open
B. Assist patient to a lying position, move away close furniture, and loosen tight clothes
C. Assist the patient to a seated position, move away close furniture, and loosen tight clothes.
D. Assist the patient to a seated position, move away close furniture, loosen tight clothing, and
prop the patient’s mouth open

23. You are working in a Direct Access environment and are evaluating a 5-year-old girl with an
elbow injury. According the mother, the injury was sustained when the mother suddenly pulled
on the girls hand to prevent her running on to a busy street. The girl is holding the arm in a
protected position and is refusing to let you move it. Which of the following is the most likely
diagnosis?

A. Nursemaid’s elbow
B. Radial head fracture
C. Child abuse
D. Muscle strain
24. You are working in a Direct Access environment and are evaluating a 5-year-old girl with an
elbow injury. According the mother, the injury was sustained when the mother suddenly pulled
on the girls hand to prevent her running on to a busy street. The girl is holding the arm in a
protected position and is refusing to let you move it. Which of the following is the most likely
diagnosis?

A. Nursemaid’s elbow
B. Radial head fracture
C. Child abuse
D. Muscle strain

25. All of the following are true statements about correct clinical documentation, except:

All documents must be legible and should be written in black or blue ink, typed and/or
transcribed
Empty lines should not be left between one entry and another, nor should empty lines be left
within a single entry
Any mistake should be crossed out with a single line through the error, initialed, and dated by
the clinician
Correction fluid/tape or similar products can be used to correct text in the medical records

26. You see a patient in the intensive care unit with multiple trauma and severe
traumatic brain injury. A chest tube is in place and it exits from the right thorax. The
patient is in need of bronchial hygiene. In this case:

A. postural drainage can be performed using the Trendelenburg position


B. percussion and shaking can be done only in the right sidelying position
C. percussion and shaking can be done carefully in the area surrounding the chest tube
D. postural drainage is permissible; however, percussion and shaking can be done only when
the chest tube is removed

27. To promote upright posture and higher walking speeds in a child with spastic
diplegia the ambulatory aid that is most beneficial is:

A. a reciprocating gait orthosis


B. an anterior rollator walker
C. a posterior rollator walker
D. a parapodium

28. In a hospital situation, OSHA has no jurisdiction regarding:

proper grounding of electrical equipment


vaccination of the staff against Hepatitis B
biohazard disposal
workplace access for handicapped employees
29. A patient with a transverse spinal cord injury has total lack of hip flexion, abduction
and knee extension. This functional picture is consistent with a designation of a
complete spinal cord lesion at the level of

A. T12/L1
B. L2/L3
C. L3/L4
D. L4/L5

30. A therapist wants to know whether joint mobilization is an effective treatment tool in
reducing joint pain in patients with degenerative joint disease. In designing a clinical
research study, joint mobilization is the:

A. control variable
B. intervening variable
C. independent variable
D. dependent variable

31.
You are performing sensory tests on a patient diagnosed with C6 nerve root
impingement. Testing should concentrate on the:

A. 3rd, 4th and 5th fingers


B. ulnar border of the hand
C. thumb and index fingers
D. medial forearm

32. The muscles that can function as synergists when acting on the scapula are the:

A. rhomboids and trapezius


B. rhomboids and serratus anterior
C. serratus anterior and levator scapulae
D. teres major and infraspinatus

33. The muscles that can function as synergists when acting on the scapula are the:

A. rhomboids and trapezius


B. rhomboids and serratus anterior
C. serratus anterior and levator scapulae
D. teres major and infraspinatus

34. As a result of a cerebellar lesion, a patient presents with incoordination and


problems with balance and posture. To help improve this situation it would be best if the
therapeutic program incorporated:
A. lower extremity splinting to improve stability and isokinetic exercises at moderate to fast
speeds to emphasize reciprocal movement
B. weight cuffs to increase proprioceptive feedback and use of rhythmic stabilization and slow
reversals-hold techniques to facilitate stability
C. Frenkel’s exercises to help with incoordination and PNF activities emphasizing traction of the
proximal joints.
D. perturbed balance using a gymnastic ball and PNF patter using D1 diagonals which promote
proximal mobility with distal stability

35. You are performing clinical research in which a specific myofascial technique is
applied to a patient with chronic neck pain. A single case study, time-series design
using A-B-A-B format is to be followed. A more in-depth follow up study would not be
justified if:

A. A equals B
B. A is greater than B
C. B is greater than A
D. BislessthanA

36. When recommending a wheelchair for a patient with a lesion at T11 resulting in
paraplegia, the modification least needed by a patient with this disability would be:

A. swing away foot rests


B. high seat back
C. pneumatic tires
D. caster locks

37. Damage as a result of Salter-Harris type IV supracondylar humeral epiphyseal


fracture in a young athlete will most likely result in:

A. refracture at a future time


B. nonunion
C. arrested growth
D. severing of the radial nerve

38. A patient is immersed up to the neck in a therapeutic pool. While exercising this
patient, the therapist should take into consideration the physiological effects of
immersion which result in significantly:

A. increased forced vital capacity


B. increased expiratory reserve volume
C. increased work of breathing
D. decreased pulmonary blood flow
39. A patient with chronic asthma has been admitted to the hospital for an acute
exacerbation of the disorder. The most important thing for the physical therapist to
check when seeing this patient for the first time is:

A. current medications the patient is taking


B. previous history of the disease relative to the patient’s current status
C. interpretation of the most recent chest x-ray
D. pulmonary function test results

40. While crossing the finish line of a 100 meter race an athlete lunged and fell on her
dorsiflexed wrist. She immediately complained of pain and tenderness over the radial
aspect of the wrist in the area of the anatomical snuffbox. You suspect a possible
fracture of the:

A. lunate
B. capitate
C. scaphoid
D. hook of the hamate

41. A patient with adhesive capsulitis of the glenohumeral joint should demonstrate the
greatest limitation of motion when performing shoulder

A. flexion.
B. abduction.
C. medial rotation.
D. lateral rotation.

42. Following a reattachment of the flexor tendons of the fingers, one physical therapy
goal is to minimize adhesion formation. A few days after surgery, with the patient in a
splint, the physical therapist should teach the patient to perform:

A. passive extension and active flexion of the interphalangeal joints


B. active extension and flexion of the interphalangeal joints
C. active extension and passive flexion of the interphalangeal joints
D. gentle passive extension and flexion of the interphalangeal joints

43. If a therapist is planning to establish a new private physical therapy practice, it is


best to first:
A. do a community needs assessment
B. outline a budget to determine how much of a business loan is required
C. seek an ideal location or building site for the practice
D. circulate a brochure informing physicians and other practitioners of this new service.
44. You are organizing a group exercise session in a therapeutic pool. Use of the pool
would be contraindicated for a group member who has:

A. an indwelling catheter
B. paraparesis and is on a regular bowel program
C. unstable blood pressure
D. an open skin lesion, even if it is covered by waterproof dressing

45. Immediately contact appropriate authorities if you:

A. suspect a patient had been driving under the influence of alcohol


B. suspect a patient has been abusing cocaine
C. notice illegal drug paraphernalia on a table next to a patient during a home visit.
D. believe a patient might be a victim of battered child syndrome

46. A patient with degenerative joint disease of the right hip complains of pain in the
anterior hip and groin, which is aggravated by weight bearing. There is decreased range
of motion and capsular mobility. Right gluteus medius weakness is evident during
ambulation and there is decreased tolerance of functional activities including transfers
and lower extremity dressing. In this case, a capsular pattern of joint motion should be
evident by restriction of hip:

A. flexion, abduction and internal rotation


B. flexion, adduction and internal rotation
C. extension, abduction and external rotation
D. flexion, abduction and external rotation

47. Confirmation of a diagnosis of spondylolisthesis can be made when viewing an


oblique radiograph of the spine. The tell-tale finding is:
*
A. posterior displacement of L5 over S1
B. bamboo appearance of the spine
C. compression of the vertebral bodies of L5 and A1
D. bilateral pars interarticularis defects

48. A boutonniere deformity of the finger would not consist of

A. hyperextension of the proximal interphalangeal joint


B. overstretch of the extensor digitorum communis tendon
C. volar slippage of the lateral bands
D. hyperextension of the distal interphalangeal joint
49. When using a patellar-tendon-bearing prosthesis, a patient will experience
excessive knee flexion in early stance if the:

A. socket is aligned too far anteriorly


B. socket is aligned too far posteriorly
C. foot is outset excessively
D. foot is inset excessively

1. A physical therapist is evaluating a patient who has a vascular lesion in the brainstem
affecting the oculomotor nerve (III). During the cranial nerve examination, which of the
following signs would be the MOST significant?
1 point

A. Inability to close the eyelid


B. Medial strabismus
C. Ptosis of the eyelid
D. Constricted pupil

2. A physical therapist is conducting a 12-minute walk test with a patient who has
chronic obstructive pulmonary disease and uses 2 L/min of oxygen by nasal cannula.
The patient’s resting oxygen saturation is 91% and resting
1 point

A. patient’s carbon dioxide level starts to increase.


B. patient starts to report shortness of breath.
C. patient’s oxygen saturation falls below 87%.
D. D. patient’s heart rate is greater than 150 bpm.

3. A patient with a complete thoracic spinal cord injury is sitting in a wheelchair on a


custom made cushion. Pressure relief activities should be performed:
1 point

A. when the patient shows signs of pressure sores.


B. every 15 to 20 minutes.
C. every 1 to 2 hours.
D. if the patient does not have an appropriate cushion

4. A therapist is measuring passive knee range of motion in a patient. The


measurements obtained are shown in photographs A and B. (A – more knee flexion with
hip flexed; B – less knee flexion with the hip extended). The MOST likely cause of the
difference in knee range of motion is:
1 point

A. knee joint capsule restriction.


B. tightness in the rectus femoris.
C. weakness of the hamstrings.
D. tightness in the vastus medialis

5. A patient who has a right piriformis syndrome is referred to physical therapy for
evaluation and intervention. The patient’s history includes a total hip arthroplasty on the
right side 2 years ago. Because of the total hip arthroplasty, which of the following
interventions require added precautions for this patient?
1 point

A. Transcutaneous electrical nerve stimulation


B. Continuous ultrasound
C. Hot packs
D. Massage to the right hip

6. A patient who has a right piriformis syndrome is referred to physical therapy for
evaluation and intervention. The patient’s history includes a total hip arthroplasty on the
right side 2 years ago. Because of the total hip arthroplasty, which of the following
interventions require added precautions for this patient?
1 point

A. Transcutaneous electrical nerve stimulation


B. Continuous ultrasound
C. Hot packs
D. Massage to the right hip

7. A patient reports anterolateral shoulder pain with an insidious onset. Examination


shows full passive range of motion pain on passive lateral (external) rotation and pain
on resistive medial (internal) rotation. These signs are consistent with a diagnosis of:
1 point

A. bicipital tendonitis
B. supraspinatus tendonitis
C. subscapularis tendonitis
D. infraspinatus tendonitis
Option 5

8. A 14 month-old child with spastic diplegia is up on the tiptoes with the toes curled
when held in supported standing. This position is characteristic of a:
1 point

A. proprioceptive placing reaction.


B. moro reflex
C. plantar grasp reflex
D. traction response
9. Following trauma at the C5 spinal cord level, a patient was admitted to the
hospital. Twenty-four hours later, the patient shows no reflexes, sensation, or voluntary
motor activity below the level of injury. These findings indicate:
1 point

A. the presence of spasticity


B. decerebrate rigidity
C. spinal shock
D. a lower motor neuron lesion

10. An initial physical therapy evaluation is performed on an elderly patient who is 1


day post total left hip arthroplasty (non-cemented) using a posterior-lateral approach.
The patient has no complicating medical history and was active and independent
preoperatively. Which of the following activities is NOT an appropriate goal for the first
week of therapy?
1 point

A. Active-assistive positioning of the left hip to 60° of flexion


B. Active, left hip abduction in right sidelying
C. Independent bed mobility with use of a trapeze
D. Walking with moderate assistance with a standard walker to 25 ft (7.6 m)

11. A physical therapist is working with a patient who is aware of being terminally ill.
What is the MOST appropriate intervention when the patient wants to talk about the
prognosis?
1 point

A. Discourage discussion of death or dying.


B. Refer the patient for pastoral counseling.
C. Relate the therapist’s experiences with other patients.
D. D. Encourage the patient’s expression of feelings.

12. A postural correction program for a patient with forward head, kyphosis, and
increased lumbar lordosis should include all of the following EXCEPT:
1 point

A. strengthening the scapular protractors.


B. strengthening the thoracic erector spinae muscles.
C. lengthening the short suboccipital muscles.
D. lengthening the lumbar erector spinae muscles.

13. A patient with a diagnosis of cervical radiculopathy reports numbness of the right
little finger (5th digit). The physical therapist will MOST likely find a diminished tendon
reflex in the:
1 point

A. biceps brachii
B. deltoid
C. triceps brachii
D. brachioradialis
Option 5

14. Following removal of a long-leg cast, a patient has limited knee flexion. The
MOST appropriate direction of patellar mobilization is:
1 point

A. distal
B. lateral
C. proximal
D. medial
Option 5

15. A patient is referred to physical therapy with a diagnosis of low back pain.
Radiographic studies, including magnetic resonance imaging, have ruled out the
presence of disc pathology. The patient reports continuous back pain that radiates
upward toward the thorax and anteriorly into the abdominal region. The physical
therapist should consider which of the following areas as a potential source of the
discomfort?
1 point

A. dura mater
B. diaphragm
C. kidney
D. urinary bladder
Option 5

16. A physical therapist reads an article on a muscle physiology study. The results of
the study are shown in the graph (length-tension curve). The therapist can BEST use
the results of the study to explain the underlying rationale for which of the following
interventions?
1 point

A. The use of prolonged passive stretching to lengthen shortened connective tissue


B. The use of plyometrics to enhance muscle power
C. The use of closed chain versus open chain exercises to enhance co-contraction of muscles
D. D. The use of hold-relax techniques for muscle stretching
17. While ascending stairs, an elderly patient leans forward with increased hip
flexion. Which of the following muscles are being used to the best advantage with this
forward posture?
1 point

A. rectus femoris
B. tensor fascia latae
C. gluteus maximus
D. erector spinae

18. The brother of a patient who was recently discharged from the hospital’s
outpatient physical therapy department telephones on the patient’s behalf to request a
copy of the patient’s medical record. The physical therapist should explain to the
patient’s brother that the medical record is the property of the:
1 point

A. patient’s family and can be released to the brother upon written request.
B. patient’s insurer now and that the request for a copy must be made in writing to the insurer.
C. hospital and the patient and can be released only with written authorization from the patient.
D. D. hospital and can be released only with written authorization from the patient’s physician.

19. To minimize skin irritation during functional electrical nerve stimulation, the
physical therapist should use: A. lower intensity, larger interelectrode distance, and
larger electrodes.
1 point

A. lower intensity, larger interelectrode distance, and larger electrodes.


B. lower intensity, larger interelectrode distance, and smaller electrodes.
C. higher intensity, smaller interelectrode distance, and smaller electrodes.
D. lower intensity, smaller interelectrode distance, and larger electrodes.

20. A patient who sustained a left transtibial amputation 2 years ago and a right
transtibial amputation 3 weeks ago is being evaluated for possible walking with
prosthesis. Which of the following factors is MOST relevant
1 point

A. Size of the right residual limb scar


B. Length of the right residual limb
C. Proficiency in previous prosthetic use
D. Severity of phantom pain

21. A patient has difficulty palpating the carotid pulse during exercise. The patient
should be instructed in alternate methods of self-monitoring, because repeated
palpation is likely to result in
1 point

A. increasing the heart rate.


B. decreasing the heart rate.
C. an irregular heart rhythm.
D. increasing systolic blood pressure.

22. A physical therapist plans to study the effect of cold compresses on passive
range of motion in a group of 10 patients. The plan is to apply these compresses to the
hamstring muscles 1 time/day for 5 days. Which of the following experimental designs is
MOST appropriate for this type of study?
1 point

A. For both the experimental and control groups, gather data from patient records.
B. For both the experimental and control groups, measure range of motion of both groups on
day 5.
C. For both the experimental and control groups, measure range of motion on days 1 and 5.
D. For the experimental group, measure range of motion every day. For the control group,
measure range of motion on days 1 and 5

23. While a patient is walking in the parallel bars, the physical therapist observes that
the pelvis drops down on the side opposite the stance extremity. This gait deviation is
an indication of weakness of the hip:
1 point

A. abductors of the swing extremity


B. adductors of the swing extremity
C. abductors of the stance extremity.
D. adductors of the stance extremity.

24. When treating a patient who has ankylosing spondylitis, the muscles requiring
the MOST emphasis for strengthening exercises are the:
1 point

A. pectorals.
B. hip flexors.
C. back extensors
D. abdominals.

25. A patient with Parkinson’s disease has just been admitted to the rehabilitation
unit. The patient is dependent in all transfers and requires moderate assistance of 1
person to walk 30 ft (9.1 m) with a standard walker. To facilitate good carry-over for
activities, instruction of the family in transfers should occur:
1 point
A. during a home visit after the patient is discharged
B. just prior to discharging the patient
C. early in the rehabilitation program
D. when the family feels ready to take the patient home

26. A patient has a history of neck pain that is aggravated by long periods of sitting.
The pain becomes progressively worse by evening. Range of motion and strength of the
neck and shoulders are within normal limits. Sensation and reflexes are intact in both
upper extremities. The patient has a forward head and excessive thoracic kyphosis. The
MOST appropriate exercise program includes:
1 point

A. stretching of the neck flexors and pectoral strengthening


B. upper trapezius strengthening and pectoral stretching.
C. pectoral strengthening and rhomboid stretching
D. rhomboid strengthening and axial neck extension

27. To achieve maximum reduction of lymphedema following a mastectomy in the


upper extremity by means of massage, it is MOST important that:
1 point

A. local heat be applied before the massage


B. the upper arm be massaged before the forearm
C. the hand be massaged before the forearm.
D. the massage strokes occur in a centrifugal direction

28. Which of the following objectives is MOST important prior to discharge, for a
patient who has had a myocardial infarction?
1 point

A. Ascend a flight of stairs before discharge


B. B. Perform prescribed exercises without angina
C. Return to normal daily activity level.
D. Take a radial pulse reliably before discharge

29. A physical therapist who works in a home health agency is treating a patient with
diabetes mellitus. The patient tells the therapist that he is no longer taking his insulin.
The physical therapist’s FIRST course of action should be to:
1 point

A. instruct the patient in the proper technique for injection of insulin


B. contact the patient’s home health nurse.
C. tell the patient’s family to report this information to the physician.
D. have the patient perform a urine glucose test while the therapist is in the home.
30. Which of the following instructions is MOST appropriate for teaching a patient
with C6 quadriplegia to transfer from a wheelchair to a mat?
1 point

A. Keep your fingers extended to give a broader base of support


B. Rotate your head and shoulders in the same direction as the desired hip motion.
C. Rotate your head and shoulders in the direction opposite to the desired hip motion.
D. Keep both hands next to your knees to lock your elbows.

31. A patient, who recently had an ankle injury, reports pain in the front of the ankle
while walking. This pain is reproduced when, with the patient in sitting, the physical
therapist grasps the patient’s heel and with the foot remaining in neutral introduces a
lateral (external) rotation force to the lower extremity. The pain is not reproduced with
inversion or eversion tests to the ankle. The patient MOST likely has which of the
following injuries?
1 point

A. Syndesmosis sprain
B. Calcaneocuboid sprain
C. 5th metatarsal fracture
D. Tibial stress fracture

32 A patient with low back and leg pain has been seen 2 times in outpatient physical
therapy. Today, during the patient’s third visit, the therapist notes an audible slap of the
foot at heel strike (initial contact). The patient also reports that there has been no leg
pain since yesterday. Regarding testing, it is MOST important for the physical therapist
to perform
1 point

A. Achilles’ tendon reflex testing.


B. sensation testing of the sole of the foot.
C. the sign of the buttock test.
D. tibialis anterior strength testing

33. A patient, who has fallen 3 times while walking on uneven surfaces, comes to
physical therapy for intervention. Examination reveals that the patient does not have
difficulty maintaining balance with the eyes closed or with the introduction of visual
conflict. Which of the following interventions is MOST appropriate for this patient?
1 point

A. Instruct the patient to walk a straight line while glancing left and right.
B. Instruct the patient in exercises to strengthen the gastroc-soleus muscle group.
C. Refer the patient to an orthotist for a custom ankle-foot orthoses fitting
D. Provide the patient with a cane for walking

34. A patient has had left Achilles’ tendinitis for 6 weeks. No structural abnormalities
are evident upon examination of the patient. Which of the following supportive shoe
inserts is MOST appropriate for this patient
1 point

A. Full-contact custom orthosis, left only


B. Off-the-shelf heel lift, left only
C. Off-the-shelf heel lifts, bilaterally
D. Full-contact custom orthoses, bilaterally

35. A physical therapist is working with an outpatient who had a cerebrovascular


accident and currently lives in an assisted-living facility. Which of the following
statements is the MOST appropriate functional goal for this patient?
1 point

A. The patient will be able to don an ankle-foot orthosis with assistance


B. The patient will independently walk 165 ft (50 m) with a straight cane from the bedroom to
the cafeteria.
C. The patient will have Normal (5/5) strength of the quadriceps muscles
D. The patient’s balance will improve to be able to independently stand on the involved lower
extremity for 20 seconds

36. A physical therapist is working on transfers with a patient who had a brainstem
cerebrovascular accident. The patient has ataxia in all 4 extremities and a high level of
extensor tone in the lower extremities. The patient has fair to good trunk control. Which
of the following transfers is BEST for this patient?
1 point

A. Squat-pivot
B. Sliding-board
C. Standing-pivot
D. Dependent tuck

37. A patient with an L4 - L5 posterolateral herniated nucleus pulposus is MOST


likely to have sensory deficits in which of the following locations?
1 point

A. Medial knee
B. Medial ankle
C. Plantar aspect of the foot
D. Dorsum of the great toe
38. A patient had a brainstem stroke 2 months ago. The patient is currently able to
independently walk 65 ft (20 m) over level surfaces with a straight cane and ascend
stairs with minimum assistance. Which of the following activities would MOST
appropriately challenge this patient’s balance during a physical therapy session?
1 point

A. Ascending stairs using a single handrail


B. Standing on 1 leg with eyes closed
C. Walking over uneven terrain
D. Walking 130 ft (40 m) with a straight cane

39. A patient who recently had bilateral leg amputations wants to have a ramp built to
travel from the back deck of his house to the pool in his wheelchair. The vertical
distance from the door to the ground level is 5 ft (1.5 m). Which of the following ramp
specifications is BEST for this patient?
1 point

A. 1 continuous ramp, 60 ft (18 m) long


B. 1 continuous ramp, 30 ft (9 m) long
C. 2 ramps, each 60 ft (18 m) long, connected by a level area
D. 2 ramps, each 30 ft (9 m) long, connected by a level area

40. The results of pulmonary function tests of a patient with which of the following
diseases is LEAST likely to show increased residual volume?
1 point

A. Atelectasis
B. Bronchiectasis
C. Chronic bronchitis
D. Emphysema

41. A patient reports numbness and tingling on the lateral side of the right lower leg
that sometimes extends into the top of the foot. Which of the following structures is
MOST likely involved?
1 point

A. Lateral sural cutaneous nerve


B. L4 nerve root
C. Superficial peroneal nerve
D. S1 nerve root

42. Which of the following outcomes is the HIGHEST expected functional outcome
for a patient with a complete C7 spinal cord injury?
1 point
A. Minimal assistance with transferring from the floor to the wheelchair
B. Independent with wheelchair mobility on smooth surfaces
C. Minimal assistance with transferring from supine to sitting
D. D. Independent with ascending a curb in the wheelchair

43. In splinting or immobilization, the functional position of the hand includes wrist:
1 point

A. extension, phalangeal flexion, and abduction of the thumb (1st digit)


B. extension, phalangeal flexion, and abduction of the thumb (1st digit)
C. extension, phalangeal flexion, and adduction of the thumb (1st digit).
D. extension, phalangeal flexion, and adduction of the thumb (1st digit

44. Which of the following assignments is MOST appropriate for a physical therapist
to delegate to a volunteer?
1 point

A. Restocking treatment booths with linens, ultrasound gel, and massage lotion
B. Attending a patient who is on a tilt table while the therapist takes a phone call
C. Transporting a patient who reports dizziness back to the patient's room
D. D. Transferring a patient from the mat table to a wheelchair

45. A client on a weight-loss program has been walking 3 days/week for 15 minutes, for
the past 3 weeks. To progress the exercise program, which of the following will MOST
likely accomplish the weight-loss goal?
1 point

A. Maintain the current walking speed and increase the duration to 30 minutes
B. Increase the walking speed and keep the duration at 15 minutes.
C. Walk 4 days/week and decrease the duration to 10 minutes.
D. Change from walking 3 days/week to jogging 1 day/week for 20 minutes.

46. During the shoulder examination of a patient, a physical therapist notes the
presence of a capsular pattern without radicular pain. To help establish the cause of the
capsular pattern, the therapist should NEXT
1 point

A. perform axial compression on the cervical spine to check for nerve root compression
B. ask the patient if there has been any prior trauma to the shoulder joint.
C. check for a painful arc during active range of motion
D. examine the shoulder for a rotator cuff tear

47. A physical therapist is assessing the lifting technique of a patient who has a
history of back pain. The patient performs a lift by picking up a light weight from the
floor, without bending the knees. During the lift, the therapist notes that the patient
demonstrates excessive lumbar flexion. Limitation of which of the following measures is
the STRONGEST contributor to this finding?
1 point

A. Hamstring flexibility
B. Gluteal muscle strength
C. Abdominal muscle strength
D. Hip flexor flexibility

48. A physical therapist is evaluating a patient who had a stroke and is exhibiting
poor foot and ankle control. When asked to lift the foot more during the midswing phase
of gait, the patient dorsiflexes the ankle with excessive eversion. Facilitatory
electromyographic biofeedback to which of the following muscles is MOST likely to help
correct this problem?
1 point

A. Extensor digitorum longus


B. Tibialis anterior
C. Fibularis (peroneus) brevis
D. Flexor hallucis longus

49. A patient presents with weakness throughout the right lower extremity with
normal strength on the left side. Sensory testing shows a loss of pressure sensation
over the right thigh and leg, and a loss of pain and temperature sensation over the left
thigh and leg. The patient has a positive Babinski’s sign present on the right. Which of
the following associated findings is MOST likely to be found during further examination
of this patient?
1 point

A. The presence of clonus in the left ankle


B. Marked atrophy in the right lower extremity muscles
C. Spasticity in the left lower extremity
D. Increased deep tendon reflexes on the right side

50. A patient was injured in a hunting accident, sustaining a gunshot wound to the
spine in the area of L1. The patient has weakness of the left lower extremity and
inability to move the knee, ankle, or foot. The patient’s patellar tendon and Achilles’
tendon reflexes are increased on the left side. There is also loss of proprioception in the
patient’s left ankle and knee, a positive Babinski’s sign on the left side, and diminished
sensation to pinprick and temperature changes in the right thigh, leg, and foot. Results
of all of the patient’s cranial nerves’ tests are normal. These findings are consistent with
which of the following injuries?
1 point

A. Complete severance of the spinal cord


B. Injury to the left anterior horn of the spinal cord
C. Injury to the left side of the spinal cord
D. Injury to the central area of the spinal cord

1. During pregnancy, which of the following exercises is CONTRAINDICATED?


1/1
A. Curl-ups
B. Bridging
C. Double leg lifts

D. Deep breathing with forced expiration


Feedback
C. During pregnancy and postpartum, the stretched abdominal muscles are unable to stabilize the
lower back as the
legs are raised. Attempting to perform double leg lifts can overwork the abdominal muscles and
cause damage to
spinal joints.

2. For a child with Duchenne muscular dystrophy, the MOST appropriate physical
therapy goal is:
1/1

A. prevention of contractures and determination of the best method of mobility

B. preservation of strength and muscle tone.


C. inhibition of abnormal muscle tone and facilitation of normal movement and postural
reactions
D. facilitation of normal movement and improvement of strength.
Feedback
A. Goals of PT intervention for a child with Duchenne muscular dystrophy are to retard the
development of
contracture and muscle weakness, which could lead to functional limitations, and thus, disability.
The physical
therapist would also play a role in determining the appropriate use of assistive devices that could
help maintain the
child’s mobility such as wheelchairs, walkers and orthoses. Muscle tone changes and declines in
strength cannot be
prevented since they are results of the disease process.
3. Outcomes of a prenatal exercise program would NOT include
1/1

A. improved body mechanics.


B. application of relaxation techniques.
C. improved ligamentous flexibility.

D. strengthened pelvic-floor musculature


Feedback
C During pregnancy, the ligaments soften due to hormonal influences, and allow some degree of
separation
between the joint surfaces. Additional stretching of the ligaments could result in joint instability or
injury, and
would not be a goal of treatment. The remaining options are all appropriate interventions.

4. To conduct an experimental study on pain in postsurgical orthopedic patients, a


physical therapist randomly assigns patients into 2 groups. One group is treated with
transcutaneous electrical nerve stimulation, heat, and exercise; the second receives
heat and exercise only. In this experimental design, transcutaneous electrical nerve
stimulation is the:
1/1

A. continuous variable.
B. dependent variable
C. discrete variable
D. independent variable.

Feedback
D The independent variable can be thought of as the cause or treatment and the dependent variable
can be thought
of as the effect or response. In this case the TENS is the treatment or independent variable.
Continuous and discrete
variables are methods of quantifying variables

5. The intervention for a patient who has right sciatic pain caused by piriformis
compression should NOT include:
1/1

A. instruction in mild self-stretching in sitting with the right hip and knee flexed and pressure
applied in the medial direction to the distal thigh with the left upper extremity
B. contract-relax exercises to the hip external rotators performed with the patient sidelying on
the left and theright hip and knee positioned in 90° of flexion
C. active resistive strengthening exercises to the piriformis with the patient prone and the knee
flexed.

D. application of cold to the area of sciatic nerve irritability.


Feedback
C The piriformis muscle functions as an external rotator of the hip, and it is thought that a tight
piriformis muscle
may compress the sciatic nerve causing pain. Passive internal rotation and resisted external rotation
may be painful.
Intervention would call for stretching of the piriformis muscle, not strengthening it. Modalities such
as ice may also
be helpful to decrease the inflammation.

6. A physical therapist is assigned the planning and implementation of physical therapy


service for all members of a community. The therapist’s FIRST step should be to:
1/1

A. develop a brochure for distribution to the community.


B. organize a health fair to provide screening for the community.
C. evaluate existing services and community resources.

D. initiate contacts in the medical community to establish prescriptive relationships.


Feedback
C The most appropriate first step (pre-planning) would be to evaluate existing services within the
community and
work out a budget for the project. Options A and B, although important, are things that would most
likely be done
after the practice is up and running. Option D may be done early on in the planning process, but
would follow
Option C.

7. A patient has right hemiparesis resulting from a traumatic brain injury. When
assessing motor control in the right lower extremity with the patient standing, the
physical therapist finds that the patient cannot extend the hip while flexing the knee or
flex the hip while extending the knee. In which of the following functional activities will
this problem be MOST apparent?
0/1
A. Shifting weight while standing
B. Walking sideways
C. Walking backward
D. Moving from a sitting position to a standing position

Correct answer
C. Walking backward

8. For a patient with insulin-dependent diabetes who is completing a cardiovascular


fitness program, what change in diabetic management is MOST likely to be instituted as
fitness increases?
1/1

A. Switching to oral rather than injected medication


B. Decreasing caloric intake for 2 to 3 hours following exercise sessions
C. Decreasing the amount of insulin taken daily

D. Increasing the amount of insulin taken daily


Feedback
C Exercise has been shown to increase sensitivity of the insulin receptors therefore leading to a
decrease in the
amount required. Administration of medication is dictated by tolerance and efficacy of the
medication and would
not necessarily be altered by exercise. In patients with moderate hyperglycemia, exercise can lead to
hypoglycemia
for periods of 24 to 48 hours after exercise, therefore increasing caloric intake, particularly
carbohydrates, would be
essential

9. A patient has disuse atrophy of the anterior compartment muscles following cast
removal after a fracture of the tibia. The patient has Poor (2/5) strength in the ankle
dorsiflexors. Electrical stimulation is to be used to enhance dorsiflexor strength in
conjunction with exercise. Initially, which of the following waveform characteristics are
MOST appropriate to stimulate the muscles?
1/1

A. Monophasic pulse with a frequency of 100 pps and an on/off cycle ratio of 3:1
B. Biphasic pulse with a frequency of 30 pps and an on/off cycle ratio of 1:5

C. Direct current waveform with an on/off ratio of 1:1


D. Interferential waveform with a beat frequency of 1 pps
Feedback
B The most efficient stimulus would be one that causes tetany in the muscle(s) without causing
fatigue. The
biphasic waveform at 30 pps best fits this stimulus. The 100 pps stimulus rate may cause fatigue,
which would be
undesirable. The interferential waveform at a frequency of 1 pps is too low and the direct current
would not be
appropriate because it would not cause tetany. In addition an on/off cycle of 1:5 would allow
adequate recovery
time for the muscles between successive contractions.

10. During an evaluation, a patient lacks 10° of passive ankle dorsiflexion. The same
degree of limitation is present whether the knee is flexed or extended. The muscle
MOST likely contributing to this restriction is the
1/1

A. gastrocnemius.
B. tibialis anterior.
C. plantaris.
D. soleus.

Feedback
D The soleus originates on the tibia and fibula and inserts onto the calcaneus, crossing only the
ankle joint.
Therefore, it will have an effect on the ankle whether the knee is flexed or extended. The
gastrocnemius and
plantaris both have origins on the femur and insert onto the calcaneus crossing both the knee and
ankle joints.
Therefore, they will be placed on stretch with the knee extended and will potentially limit ankle
dorsiflexion to a
greater extent with the knee extended. The tibialis anterior is a dorsiflexor of the ankle and while
weakness in this
muscle may limit active dorsiflexion, it would not affect passive dorsiflexion.

11. A physical therapist is testing the deep tendon reflex of a patient, as shown in the
photograph. The patient has a partial nerve injury to the tested nerve root. Which of the
following reflex grades is the therapist MOST likely to find in the patient?
1/1

A. 0
B. 1 +

C. 2 +
D. 3 +
12. A patient is receiving mechanical intermittent cervical traction with an on/off duty
cycle of 20 sec/10 sec. The patient reports increased pain each time the traction unit
cycles on, which then subsides over the duration of the on time. Which of the following
modifications to the duty cycle is MOST appropriate?
0/1

A. Increase the off time to 20 seconds.


B. Increase the on time to 20 seconds.
C. Decrease the off time to 5 seconds.
D. Decrease the on time to 5 seconds.

Correct answer
A. Increase the off time to 20 seconds.

13. A patient with chronic obstructive pulmonary disease is starting to participate in a


supervised exercise program. The MOST important aspect to monitor during this
exercise program is the:
0/1
A. cardiorespiratory endurance training.

B. resistive exercise.
C. active soft-tissue stretching.
D. warm-up and cool-down intensity.

Correct answer
D. warm-up and cool-down intensity.

14. A patient with a traumatic brain injury is being discharged to home following
completion of inpatient rehabilitation. Which of the following assessment tools would
BEST assess the patient’s potential?
1/1

A. Glasgow Outcome Scale


B. Fugl-Meyer Assessment
C. Rancho Los Amigos Levels of Cognitive Functioning Scale

D. Sickness Impact Profile


15. A physical therapist is evaluating the cranial nerves of a child who has a
medulloblastoma. The child’s right eye deviates medially. This child has impairment of
which of the following cranial nerves?
1/1

A. Oculomotor (III)
B. Trochlear (IV)
C. Abducens (VI)

D. Vagus (X)

16. A patient comes to physical therapy for intervention for an ulcer on the right medial
malleolus. Upon inspection of the patient’s legs, the physical therapist notes normal skin
temperature and brownish pigmentation of the skin. The patient MOST likely also has:
0/1
A. edema.
B. diminished pedal pulse.
C. lower extremity pain with exercise.

D. hypersensitivity to cold.

Correct answer
A. edema.

17. A physical therapist is preparing for gait training with a young, adult patient with
paraplegia. Which of the following gait training options is MOST appropriate for the
patient’s first session?
1/1

A. Swing-through gait pattern with a walker


B. Swing-through gait pattern with forearm crutches
C. Swing-to gait pattern with axillary crutches
D. Swing-to gait pattern in the parallel bars

18. A 22 year-old patient is hospitalized awaiting a lung transplant due to cystic fibrosis.
The patient’s physician is interested in an objective measure of the patient’s
preoperative endurance. Which of the following tests is MOST appropriate for the
physical therapist to administer to this patient?
1/1

A. VO2 max treadmill test


B. 2-step test
C. Submaximal exercise test on a cycle ergometer
D. 6-minute walk test

19. A patient comes to physical therapy via direct access for evaluation of insidious
shoulder pain. Upon inspection, the physical therapist notes a yellowish color of the
patient’s sclera and skin. The therapist should refer this patient to a physician for
probable primary dysfunction of the:
1/1
A. liver.

B. eye.
C. duodenum.
D. heart.

20. During which of the following scenarios are gloves required to comply with standard
precautions?
1/1

A. During all patient care in the hospital setting


B. Performing range of motion on a patient with acquired immune deficiency syndrome
C. Massaging the neck of a patient with hepatitis C
D. Changing an infant’s diaper in the pediatric setting

21. A physical therapist is conducting a reflex text as shown in the photograph. The
starting and ending position of the test is indicated by the arrow. The results of the test
are MOST likely to indicate:
1/1

A. a peripheral nerve injury.


B. a lesion of the anterior horn cells.
C. a normal response.
D. an injury to the spinal cord.
22. A patient with idiopathic pulmonary fibrosis completed a 6-minute walk test and
demonstrates the following results: Total Walking Distance - 1200 ft (366 m) in 6
minutes; Heart Rate - 82 to 110 bpm (pre-test to post-test); Blood Pressure - 125/80 to
145/85 mm Hg (pre-test to post-test); Respiratory Rate - 18 to 40 (pre-test to post-test);
Oxygen Saturation - 98% to 92% (pre-test to post-test); Electrocardiogram - Normal
sinus rhythm throughout test. Based on these results, the physical therapist should
determine that this patient has impaired:
0/1

A. aerobic capacity and endurance associated with cardiovascular pump dysfunction.

B. ventilation, respiration, and aerobic capacity associated with airway clearance dysfunction.
C. ventilation, respiration, aerobic capacity and gas exchange associated with ventilatory pump
dysfunction.
D. aerobic capacity and endurance associated with cardiovascular pump failure.

Correct answer
C. ventilation, respiration, aerobic capacity and gas exchange associated with ventilatory pump
dysfunction.

23. A patient had a split-thickness skin graft for a superficial partial-thickness burn injury
to the upper extremity. The surgeon has requested range of motion exercises for the
patient. Currently, the patient is able to actively move the upper extremity through one-
third of range of motion for shoulder flexion. Based on this finding, what is the MOST
appropriate action for the physical therapist to take at this time?
1/1
A. Defer any range of motion exercises until the patient is able to participate more actively.
B. Begin active assistive range-of-motion exercises.

C. Begin bed mobility training to facilitate increased use of the upper extremity.
D. Continue with active range-of-motion exercises.

24. A 50 year-old patient with metabolic (insulin resistance) syndrome comes to


physical therapy to initiate a conditioning program. Which of the following initial exercise
prescriptions is MOST appropriate for this patient?
0/1

A. Intensity: Heart rate – 74 to 80 bpm. Frequency: 4 to 5 days/week. Duration: 30 minutes.


Mode: walking
B. Intensity: Heart rate – 105 to 115 bpm. Frequency: 2 to 3 days/week. Duration: 20 minutes.
Mode: walking.

C. Intensity: Rate of Perceived Exertion – 6 to 8 (6 to 20 scale). Frequency: 4 to 5 days/week.


Duration: 30 minutes. Mode: stationary bike
D. Intensity: Rate of Perceived Exertion – 13 to 15 (6 to 20 scale). Frequency: 2 to 3 days/week.
Duration: 20 minutes. Mode: stationary bike

Correct answer
A. Intensity: Heart rate – 74 to 80 bpm. Frequency: 4 to 5 days/week. Duration: 30 minutes.
Mode: walking

25. Upon auscultation, a physical therapist hears crackles over the third intercostal
space and right midclavicular line. To MOST appropriately document this finding, what
region of the lung should be identified by the therapist?
0/1

A. Anterior segment of the right upper lobe


B. Superior segment of the right middle lobe

C. Apical segment of the right upper lobe


D. Inferior segment of the right middle lobe

Correct answer
A. Anterior segment of the right upper lobe

26. During an initial evaluation, which of the following tests is MOST appropriate to
perform with a patient who has acute right-sided congestive heart failure?
1/1

A. Sensory testing of upper extremities


B. Pitting edema measurements in the lower extremities

C. Resisted manual muscle testing of all extremities


D. Reflex testing of lower extremities

27. A patient with Parkinson’s disease demonstrates shortness of breath with activity.
Which of the following tests is MOST appropriate for the physical therapist to perform to
help delineate a cause for the patient’s shortness of breath?
0/1
A. Deep tendon reflex testing
B. Sensory examination
C. Muscle strength testing

D. Posture examination

Correct answer
D. Posture examination

28. A patient with chest pain from myocardial ischemia will MOST likely exhibit:
0/1

A. increased pain upon chest-wall palpation.


B. increased pain with deep breathing.
C. relief with nitroglycerin (Nitrostat) ingestion.
D. relief with antacid ingestion

Correct answer
C. relief with nitroglycerin (Nitrostat) ingestion.

29. A physical therapist is using a stethoscope to auscultate a patient’s S1 and S2 heart


sounds. Of the following procedures, the MOST appropriate for the physical therapist to
use is to apply the:
1/1

A. diaphragm of the head of the stethoscope firmly on the skin.

B. bell of the head of the stethoscope firmly on the skin.


C. diaphragm of the head of the stethoscope lightly on the skin.
D. bell of the head of the stethoscope lightly on the skin.

30. While working in a private practice clinic, a physical therapist observes a patient fall
in the parking lot outside the office. The patient sustains a severe laceration to the
forearm. The physical therapist secures a pressure dressing to the wound site, but
notes that blood is soaking through the dressing and the bandage. Which of the
following actions should the therapist perform NEXT?
1/1

A. Elevate the limb and apply pressure to the wound.

B. Remove the dressing and bandage and start over with a tighter bandage.
C. Apply additional dressings and bandages and apply pressure to the brachial artery.
D. Call the patient’s physician and arrange transportation for medical care.

31. A physical therapist is developing a gait training program for a patient following a
total hip arthroplasty of the right hip done via an anterolateral approach 2 weeks ago.
The proper instructions and rationale for crutch training utilizing 1 crutch include holding
the crutch in the:
1/1

A. right hand to decrease activity in the right hip abductors.


B. right hand to facilitate activity in the right hip abductors.
C. left hand to decrease activity in the right hip abductors

D. left hand to facilitate activity in the right hip abductors


Feedback
C With an anterolateral approach, the gluteus medius is reflected or the trochanter is taken down.
Post operatively,
these structures need time to heal and will not be healed only 2 weeks after surgery. The abductors
should not be
aggressively strengthened with exercise

32. A patient received a gunshot wound to the distal posterior thigh that resulted in
complete severance of the common fibular (peroneal) nerve. Which of the following
exercises is necessary in the early rehabilitation period to prevent contracture
formation?
1/1

A. Active plantarflexion, inversion, and toe extension


B. Passive dorsiflexion, eversion, and toe extension

C. Active dorsiflexion, eversion, and toe flexion


D. Passive plantarflexion, eversion, and toe flexion
Feedback
B The common peroneal nerve innervates the following muscles: peroneus longus and brevis, tibialis
anterior,
extensor digitorum longus and brevis, peroneus tertius and extensor hallucis longus. Because of the
complete nerve
severance, all of those muscles would be affected and would not be able to actively contract until
regeneration has
taken place. Therefore, the patient would completely lose ankle dorsiflexion, eversion, and toe
extension. Those
motions would have to be performed passively in order to maintain range of motion. Option B is the
only one that
contains all of the correct actions

33. A patient with a mild closed head injury and bilateral femur fractures requires
instruction in a lower extremity exercise program. To plan the most effective teaching
methods for this patient, what is MOST critical to assess at the initial visit?
1/1

A. Comprehension of written, verbal, and demonstrated instructions

B. Short-term memory capacity


C. Auditory and visual status
D. Any personality changes compared to the patient’s premorbid status
Feedback
A Option A is the best answer. In Option B, if the patient does not initially understand the information,
then
having short-term memory is irrelevant. For Option C, hearing and vision are obviously important, but
the most
critical component is comprehension. Option D is irrelevant to the scenario.

34. For a patient with a bilateral transfemoral amputation to maximize balance in a


wheelchair, the rear wheels should be positioned more:
0/1

A. laterally.

B. posteriorly.
C. anteriorly.
D. inferiorly.

Correct answer
B. posteriorly.

35. Prior to starting an exercise training program, a patient with cardiac problems who is
receiving beta-blocking medication should receive an explanation of the:
0/1

A. greater benefits from cardiovascular exercise to be achieved at lower rather than at higher
metabolic levels.
B. need to use measures other than heart rate to determine intensity of exercise
C. need for exercise training sessions to be more frequent but of shorter duration
D. need for longer warm-up periods and cool-down periods during exercise sessions.

Correct answer
B. need to use measures other than heart rate to determine intensity of exercise

36. A patient with chronic adhesive capsulitis of the shoulder is to be treated with joint
mobilization to increase joint range of motion. Which of the following is the MOST
appropriate to increase shoulder abduction?
1/1

A. Distraction with inferior glide

B. Anterior glide with internal rotation


C. Superior glide with approximation
D. Distraction with posterior glide
Feedback
A Since most of the joint contracture/adhesion forms in the anterior-inferior portion of the capsule,
treatment to
mobilize that portion of the capsule is of primary concern. Inferior glide would stretch the inferior
portion of the
capsule

37. A physical therapist is screening a young adult patient for a possible right thoracic,
left lumbar structural scoliosis. The postural deviation commonly seen with this
diagnosis is:
0/1

A. spinous processes of thoracic spine rotated to the right


B. high left shoulder.

C. posteriorly protruding scapula on the left.


D. prominent right posterior rib cage.

Correct answer
D. prominent right posterior rib cage.

38. A physical therapist is planning a patient education program for a group of patients
with chronic low back pain. To increase the likelihood that the patients will utilize the
proper body mechanics at the work site, the therapist should:
1/1
A. ask the patients to demonstrate use of proper body mechanics.

B. provide a reference list of articles describing body mechanics


C. ask the patients to describe actions they do that increase their back pain.
D. provide information on the frequency of low back injuries due to improper body mechanics
Feedback
A Evaluation of the patient’s retention of the information presented in the program can be enhanced
by asking the
patients questions about the program information, having the patients ask questions about the
program, having the
patients demonstrate what they have learned and testing the patient about the program material.
Option A is the
only one that utilizes one of these techniques. The other options do not require active participation
by the patients,
and they do not address specific interventions

39. A physical therapist is treating a patient with iontophoresis. If the current is set at 4
mA, the duration at 15 minutes, and the dexamethasone (Decadron) solution at 0.04%,
with an electrode area of 4 cm2, how should the therapist record the current dosage in
the patient’s chart?
1/1

A. 0.16 mA-mg
B. 16 mA/cm2
C. 0.60 mg/min
D. 60 mA-min

Feedback
D The current dosage for iontophoresis is calculated by multiplying the current (4 mA) by the
duration (15
minutes). In this case the calculation would yield a value of 60 mA-min. None of the other options
use the correct
values.

40. An elderly patient has a diabetic neuropathy. Upon examination, the patient shows
marked mediolateral instability of the left ankle while walking. The patient also has
fluctuating edema and glove-and-stocking sensory loss in both of the lower extremities.
The MOST appropriate orthotic aid is:
0/1
A. a double-upright metal ankle-foot orthosis.
B. high-top shoes.
C. a prefabricated plastic solid ankle-foot orthosis
D. a spiral ankle-foot orthosis.

Correct answer
A. a double-upright metal ankle-foot orthosis.

41. A physical therapist is examining a patient who has difficulty making a fist. Results
of the patient’s muscle tests show the following: Flexors of the index, middle, ring, and
little fingers (2nd to 5th digits) at the proximal interphalangeal joint are Normal (5/5).
Flexors of the index finger (2nd) and middle finger (3rd digit) at the distal
interphalangeal joint are Normal (5/5). Flexors of the ring finger (4th digit) and little
finger (5th digit) at the distal interphalangeal joint are Poor (2/5).These findings are
consistent with a lesion of the:
1/1

A. median nerve C8-T1.


B. median nerve C5-C6.
C. ulnar nerve C5-C6.
D. ulnar nerve C8-T1.

Feedback
D The weakness is in the flexors of the fourth and fifth digits at the distal interphalangeal joint (2/5).
The other
fingers show normal strength (5/5). This weakness would indicate a problem with the flexor
digitorum profundus,
since the distal joints are involved. The flexor digitorum profundus is innervated by both the median
and ulnar
nerves. The flexor digitorum profundus muscles flexing the index and middle fingers are innervated
by the median
nerve. The flexor digitorum profundus muscles flexing the ring and little fingers are innervated by the
ulnar nerve.
Therefore, since the ring and little fingers are involved, the lesion would have to be involving the
ulnar nerve. The
flexor digitorum profundus to the fourth and fifth fingers is innervated by the ulnar nerve roots C8
and T1.

42. A physical therapist is treating a patient with moderately well-controlled type 1


diabetes. One of the goals is to help the patient regulate insulin dosage through
exercise. In establishing an exercise program, the therapist should be aware that
regular exercise will generally:
1/1

A. lower blood glucose levels and decrease the amount of insulin required.

B. lower blood glucose levels and increase the amount of insulin required
C. raise blood glucose levels and decrease the amount of insulin required
D. raise blood glucose levels and increase the amount of insulin required.
Feedback
A In patients with moderate hyperglycemia, exercise can lead to hypoglycemia for periods of 24 to
48 hours after
exercise. Exercise has been shown to increase sensitivity of the insulin receptors therefore leading
to a decrease in
the amount of insulin required

43. During inspiration, a patient demonstrates increased upper chest expansion with
retraction of the epigastric area. The physical therapist should suspect weakness of the:
0/1

A. scalene muscles.

B. diaphragm.
C. rectus abdominis.
D. intercostal muscles.

Correct answer
B. diaphragm.

44. A patient with low back pain has been undergoing treatment for 2 sessions. The
patient tells the physical therapist that today the pain is centralizing with the extension
exercises, but is as intense as it was at the first treatment session. The patient is
frustrated by this reaction. The MOST appropriate response of the therapist is to :
0/1

A. continue with the present program.


B. eliminate the extension exercises
C. consult the patient’s physician about the situation

D. progress to trunk flexion exercises

Correct answer
A. continue with the present program.
45. Involving interdisciplinary team members in making decisions about continuity of
patient care has all of the following benefits EXCEPT:
0/1

A. promoting each individual’s commitment to, and responsibility for, the decisions that have
been made by the
group
B. discouraging 1 or 2 team members from being in control and having all the authority.

C. enabling decisions regarding quality of care to be made quickly.


D. promoting quality of care by including several aspects of the patient’s care.

Correct answer
C. enabling decisions regarding quality of care to be made quickly.

46. While examining a patient who had a baby 3 days ago by vaginal delivery, the
physical therapist notices that the patient has a 2-cm diastasis of the rectus abdominis.
To address this problem, the patient should:
1/1
A. perform partial sit-ups supporting her abdominal muscles with her hands, while lifting her
head in the supine

position.
B. not perform exercises, until the diastasis heals spontaneously.
C. perform sit-ups with the knees bent and arms behind the head.
D. perform partial sit-ups with legs straight and arms in front.
Feedback
A. Raising just the head activates only the recti muscles, and supporting the abdominal muscles with
the hands
provides external support to the stretched abdominal muscles. This is the optimal position to initiate
abdominal
strengthening following delivery for a patient with diastasis rectus abdominus. Doing no exercise
would not be
appropriate. With no support of the abdominal muscles, Options C and D would be too aggressive at
this time.

47. When evaluating wheelchair positioning of a child with cerebral palsy, the physical
therapist should FIRSTexamine the position of the child’s:
1/1
A. pelvis.
B. lower extremities.
C. head.
D. spine.
Feedback
A The assessment of posture in a wheelchair begins with the pelvis and its relationship to its
adjacent segments.
The orientation and range of mobility of the pelvis in all three planes will in turn determine the
alignment and
support needed at the trunk, head, and extremities

48. For a patient with a right-middle-lobe pneumonia, the proper bronchial drainage
position is supine with the body:
0/1

A. one-quarter turned to the left and the foot of the bed raised 14 in (35.6 cm).
B. one-quarter turned to the right and the bed flat.
C. three-quarters turned to the left and the head of the bed raised 14 in (35.6 cm).
D. three-quarters turned to the right and the foot of the bed raised 14 in (35.6 cm).

Correct answer
A. one-quarter turned to the left and the foot of the bed raised 14 in (35.6 cm).

49. A patient who received an organ transplantation 4 years ago demonstrates


progressive weakness and is referred to physical therapy for strengthening exercises. In
reviewing the medical history, the physical therapist learns that the patient takes
prednisone (Deltasone) and immunosuppressive drugs. Which of the following does the
therapist need to consider?
0/1
A. Isokinetic strengthening will be preferable for this patient

B. The patient may have limited range of motion from increased bone mass.
C. The patient will need to be treated in an isolation room.
D. Muscle strengthening may be limited in this patient.

Correct answer
D. Muscle strengthening may be limited in this patient.
50. A physical therapist is treating a patient who has been diagnosed with a nerve root
impingement on his right side due to a narrowing of the intervertebral foramen between
L4 and L5. The therapist decides to use mechanical, lumbar traction as an intervention.
Which of the following positions, on a traction table, is MOST appropriate to relieve
pressure on the nerve root?
1/1

A. Supine with the hips and knees flexed

B. Supine with the hips and knees straight and laterally shifted to the right
C. Prone with the hips and knees straight
D. Sidelying on the right with a bolster between the table and patient
Feedback
A In order to open up the intervertebral foramen, it would be best to flex or at least flatten the lumbar
spine prior
to applying the traction. Of the above options, positioning the patient supine with the hips and knees
flexed would
be the best way to do this. Laterally bending the trunk to the left would also open up the foramen, but
this is not one
of the options. Supine with hips and knees straight and then laterally shifted to the right would tend
to close down
the intervertebral foramen. Prone would also close the foramen. Sidelying with a bolster under the
right lumber
spine would also tend to close the foramen

1. A physical therapist is evaluating a patient who has a vascular lesion in the brainstem
affecting the oculomotor nerve (III). During the cranial nerve examination, which of the
following signs would be the MOST significant?
1/1

A. Inability to close the eyelid


B. Medial strabismus
C. Ptosis of the eyelid

D. Constricted pupil
Feedback
C. The oculomotor nerve innervates the levator palpebrae superioris muscle that elevates the upper
eyelid and the pupillary constrictor muscle. Therefore, a lesion of the oculomotor nerve would make
it difficult, if not impossible, for the patient to fully raise the lid (open the eye) and would cause a
condition termed ptosis. In addition, the eye may not react to light and therefore would not show
pupillary constriction when light is directed into the eye. Inability to fully close the eye would be seen
with a lesion of the facial nerve (Bell’s palsy). Medial strabismus would be caused by damage to the
abducens nerve, innervating the lateral rectus causing a medial strabismus.
2. A physical therapist is conducting a 12-minute walk test with a patient who has
chronic obstructive pulmonary disease and uses 2 L/min of oxygen by nasal cannula.
The patient’s resting oxygen saturation is 91% and resting
1/1

A. patient’s carbon dioxide level starts to increase.


B. patient starts to report shortness of breath.
C. patient’s oxygen saturation falls below 87%.

D. D. patient’s heart rate is greater than 150 bpm.


Feedback
C. A fall in oxygen saturation below 87 % is equivalent to a partial pressure of 55 mm Hg of oxygen
in the blood, which is considered to be moderately hypoxemic (low oxygen levels). This situation
would require increased oxygen levels in order to be rectified. A rise is carbon dioxide level would
not be alleviated by increased oxygen levels. Complaints of shortness of breath can come from a
variety of causes and would not necessarily be alleviated by increased oxygen levels. An increase in
HR to 150 bpm may be a normal response to this activity and would not necessarily require
increased oxygen levels.

3. A patient with a complete thoracic spinal cord injury is sitting in a wheelchair on a


custom made cushion. Pressure relief activities should be performed:
0/1

A. when the patient shows signs of pressure sores.


B. every 15 to 20 minutes.
C. every 1 to 2 hours.
D. if the patient does not have an appropriate cushion

4. A therapist is measuring passive knee range of motion in a patient. The


measurements obtained are shown in photographs A and B. (A – more knee flexion with
hip flexed; B – less knee flexion with the hip extended). The MOST likely cause of the
difference in knee range of motion is:
1/1

A. knee joint capsule restriction.


B. tightness in the rectus femoris.

C. weakness of the hamstrings.


D. tightness in the vastus medialis
Feedback
B. Capsular restriction would show up in both measurements. In photograph A, there is more knee
flexion present with the hip flexed. In this position the rectus femoris is on slack across the hip joint
allowing greater range of knee flexion. In photograph B the rectus femoris is stretched over both the
knee joint and the hip joint, so tightness in the rectus femoris would restrict knee flexion.
Photograph B also shows hip joint flexion. Hamstring weakness would not affect passive range of
motion. Vastus medialis tightness would affect both measurements.

5. A patient who has a right piriformis syndrome is referred to physical therapy for
evaluation and intervention. The patient’s history includes a total hip arthroplasty on the
right side 2 years ago. Because of the total hip arthroplasty, which of the following
interventions require added precautions for this patient?
1/1
A. Transcutaneous electrical nerve stimulation
B. Continuous ultrasound

C. Hot packs
D. Massage to the right hip
Feedback
B. The only one of the above interventions that requires precaution because of the total hip
replacement is
continuous ultrasound. However, that does not mean that ultrasound is contraindicated for this
patient.
Transcutaneous electrical nerve stimulation may be used over metal implants. Hot packs and
massage would not
affect the total hip prosthesis.

6. A patient who has a right piriformis syndrome is referred to physical therapy for
evaluation and intervention. The patient’s history includes a total hip arthroplasty on the
right side 2 years ago. Because of the total hip arthroplasty, which of the following
interventions require added precautions for this patient?
1/1

A. Transcutaneous electrical nerve stimulation


B. Continuous ultrasound

C. Hot packs
D. Massage to the right hip
Feedback
B. The only one of the above interventions that requires precaution because of the total hip
replacement is
continuous ultrasound. However, that does not mean that ultrasound is contraindicated for this
patient.
Transcutaneous electrical nerve stimulation may be used over metal implants. Hot packs and
massage would not
affect the total hip prosthesis.

7. A patient reports anterolateral shoulder pain with an insidious onset. Examination


shows full passive range of motion pain on passive lateral (external) rotation and pain
on resistive medial (internal) rotation. These signs are consistent with a diagnosis of:
1/1

A. bicipital tendonitis
B. supraspinatus tendonitis
C. subscapularis tendonitis

D. infraspinatus tendonitis
Option 5
Feedback
C. Pain with resisted medial (internal) rotation and pain with passive lateral (external) rotation is
indicative of
subscapularis tendonitis. Bicipital tendonitis is suspected if resisted supination is painful when the
patient’s arm is
at the side and the elbow is flexed to 90°. Painful resisted abduction and resisted lateral (external)
rotation is
indicative of supraspinatus tendinitis. Pain on resisted lateral (external) rotation is indicative of
infraspinatus
tendonitis.

8. A 14 month-old child with spastic diplegia is up on the tiptoes with the toes curled
when held in supported standing. This position is characteristic of a:
1/1

A. proprioceptive placing reaction.


B. moro reflex
C. plantar grasp reflex

D. traction response
Feedback
C. The plantar grasp reflex is characterized by curling of the toes when a child is held supported in
standing. The
reflex is normal up to 9 months of age. Delayed integration of this reflex can result in delayed,
independent
ambulation
9. Following trauma at the C5 spinal cord level, a patient was admitted to the
hospital. Twenty-four hours later, the patient shows no reflexes, sensation, or voluntary
motor activity below the level of injury. These findings indicate:
1/1

A. the presence of spasticity


B. decerebrate rigidity
C. spinal shock

D. a lower motor neuron lesion


Feedback
C. Spinal shock occurs as a reaction to spinal cord injury and is characterized by an absence of all
reflex activity
below the level of the lesion. Depending on the extent of the lesion, the patient may lose all or some
of their
sensation and motor activity below the level of the lesion. Spasticity is associated with hyperreflexia
and increased
muscle tone. Spasticity would be expected to develop following the spinal shock stage. Decerebrate
rigidity
involves a sustained contraction of the upper and lower extremities in extension. With a lower motor
neuron
lesion, the loss of sensation and motor activity would be confined to a much smaller region
(depending on the
exact lesion) and would not affect all levels below the lesion.

10. An initial physical therapy evaluation is performed on an elderly patient who is 1


day post total left hip arthroplasty (non-cemented) using a posterior-lateral approach.
The patient has no complicating medical history and was active and independent
preoperatively. Which of the following activities is NOT an appropriate goal for the first
week of therapy?
1/1

A. Active-assistive positioning of the left hip to 60° of flexion


B. Active, left hip abduction in right sidelying

C. Independent bed mobility with use of a trapeze


D. Walking with moderate assistance with a standard walker to 25 ft (7.6 m)
Feedback
B. Although protocols depend on the surgeon and the approach, it is generally recommended that
anti-gravity hip
abduction exercises not begin until 5 to 6 weeks post surgery. Patients are taught to avoid excessive
hip flexion,
usually beyond 80°. Bed mobility and ambulation would be started 1 or 2 days post operatively.

11. A physical therapist is working with a patient who is aware of being terminally ill.
What is the MOST appropriate intervention when the patient wants to talk about the
prognosis?
1/1

A. Discourage discussion of death or dying.


B. Refer the patient for pastoral counseling.
C. Relate the therapist’s experiences with other patients.
D. D. Encourage the patient’s expression of feelings.

Feedback
D. Patients should be encouraged to express their feelings. Comparisons to other patients who are
dying, in an
effort to assure the patient he is not alone, takes away from this patient’s feelings. Denial of death
would not be
good for the patient, since he must ultimately cope with the inevitable. Pastoral counseling would be
an option, but
the therapist should be ready to listen to the patient, encourage expression of feelings and avoid
denial

12. A postural correction program for a patient with forward head, kyphosis, and
increased lumbar lordosis should include all of the following EXCEPT:
1/1

A. strengthening the scapular protractors.

B. strengthening the thoracic erector spinae muscles.


C. lengthening the short suboccipital muscles.
D. lengthening the lumbar erector spinae muscles.
Feedback
A. With this particular posture, the patient’s scapula would be in a protracted (abducted) position;
therefore the
scapular protractors are already overactive and would require stretching not strengthening. The
kyphosis suggests
that the thoracic erector spinae muscles are weak and need strengthening. The lumbar lordosis
indicates shortened
lumbar erector spinae muscles. The forward head posture suggests that the cervical spine is flexed
and the occiput
is extended, therefore stretching of the suboccipital muscles would be indicated. A TEST-TAKING
HINT:
Although EXCEPT questions are rarely used on the NPTE, be alert for this type of question and read
the responses
so that you select the unrelated response.

13. A patient with a diagnosis of cervical radiculopathy reports numbness of the right
little finger (5th digit). The physical therapist will MOST likely find a diminished tendon
reflex in the:
1/1

A. biceps brachii
B. deltoid
C. triceps brachii

D. brachioradialis
Option 5
Feedback
C. The dermatome providing sensation to the little finger is innervated by the C8 nerve root. The
triceps brachii is
the only one of the muscles listed that is also innervated by the C8 nerve root. The biceps brachii,
deltoid and
brachioradialis are innervated by the C5, C6, and sometimes C7 nerve roots

14. Following removal of a long-leg cast, a patient has limited knee flexion. The
MOST appropriate direction of patellar mobilization is:
0/1

A. distal
B. lateral
C. proximal
D. medial
Option 5

15. A patient is referred to physical therapy with a diagnosis of low back pain.
Radiographic studies, including magnetic resonance imaging, have ruled out the
presence of disc pathology. The patient reports continuous back pain that radiates
upward toward the thorax and anteriorly into the abdominal region. The physical
therapist should consider which of the following areas as a potential source of the
discomfort?
1/1

A. dura mater
B. diaphragm
C. kidney
D. urinary bladder
Option 5
Feedback
C. Pain that is experienced in the thoracic spine can be caused by a variety of pathologic conditions.
Low back
pain can be either mechanical or non-mechanical in nature. Pathology in the kidney may refer pain to
the lumbar
spine (ipsilateral flank), or upper abdomen. Pathology in the urinary bladder refers to the suprapubic
or
thoracolumbar region. The diaphragm is innervated by C3, C4, and C5 with the pain normally
confined to the C4
dermatome. The negative MRI has ruled out possible involvement of the dura mater

16. A physical therapist reads an article on a muscle physiology study. The results of
the study are shown in the graph (length-tension curve). The therapist can BEST use
the results of the study to explain the underlying rationale for which of the following
interventions?
1/1

A. The use of prolonged passive stretching to lengthen shortened connective tissue


B. The use of plyometrics to enhance muscle power
C. The use of closed chain versus open chain exercises to enhance co-contraction of muscles
D. D. The use of hold-relax techniques for muscle stretching
Feedback
B. Prolonged stretching does not depend on the length tension curve, but on the stress relaxation
curve and Golgi
tendon organ. Plyometrics is a form of exercise designed to enhance muscle speed and power. The
theory behind
plyometrics relies on the stretch reflex (active contraction of a muscle or active tension) and the
natural elastic
components of skeletal muscle and its surrounding connective tissue (passive tension). The graph
shows that as the
muscle is stretched, the passive components are stretched and add to the total amount of tension
that can be
developed in a muscle. During plyometric exercises, a muscle is quickly stretched activating the
muscle spindle
and the stretch reflex, as well as stretching the connective tissue elements surrounding the muscle
fibers. This
combination produces a quicker, more forceful, contraction by the muscle. Closed chain exercise
use is not
explained by the length tension curve. The hold relax technique is thought to be mediated via the
Golgi tendon
organ, not length tension.
17. While ascending stairs, an elderly patient leans forward with increased hip
flexion. Which of the following muscles are being used to the best advantage with this
forward posture?
1/1

A. rectus femoris
B. tensor fascia latae
C. gluteus maximus
D. erector spinae
Feedback
C. The gluteus maximus is a hip extensor that is more active during resisted motions or when the hip
is in flexion,
especially in functional activities such as stair climbing. In this case, the patient flexes the hip
placing the gluteus
maximus on stretch increasing its ability to produce tension. Therefore, the patient is leaning
forward to maximize
the ability of the gluteus maximus to extend the hip during the stair climbing activity. The other
muscles listed
would not benefit as much from the increased hip flexion.

18. The brother of a patient who was recently discharged from the hospital’s
outpatient physical therapy department telephones on the patient’s behalf to request a
copy of the patient’s medical record. The physical therapist should explain to the
patient’s brother that the medical record is the property of the:
1/1

A. patient’s family and can be released to the brother upon written request.
B. patient’s insurer now and that the request for a copy must be made in writing to the insurer.
C. hospital and the patient and can be released only with written authorization from the patient.
D. D. hospital and can be released only with written authorization from the patient’s physician.
Feedback
C. The medical record is owned by the hospital subject to the patient’s interest in the information it
contains.
Unless restricted by state or federal law or regulation, a hospital shall furnish to a patient, or a
patient’s
representative parts of the hospital record upon request in writing by the patient or their
representative. Option C is
the only correct answer in this case, since it specifies that the patient must authorize the release of
information.
19. To minimize skin irritation during functional electrical nerve stimulation, the
physical therapist should use: A. lower intensity, larger interelectrode distance, and
larger electrodes.
1/1

A. lower intensity, larger interelectrode distance, and larger electrodes.


B. lower intensity, larger interelectrode distance, and smaller electrodes.
C. higher intensity, smaller interelectrode distance, and smaller electrodes.
D. lower intensity, smaller interelectrode distance, and larger electrodes.
Feedback
A. Several things can be done to decrease the current density and the possibility of skin irritation.
These include
decreasing the intensity of the stimulation, increasing the inter-electrode distance and the use of
larger electrodes.

20. A patient who sustained a left transtibial amputation 2 years ago and a right
transtibial amputation 3 weeks ago is being evaluated for possible walking with
prosthesis. Which of the following factors is MOST relevant
1/1

A. Size of the right residual limb scar


B. Length of the right residual limb
C. Proficiency in previous prosthetic use
D. Severity of phantom pain
Feedback
C. The previously amputated left limb must function as the main support limb. Any treatment
strategy for
ambulation must ensure that the left limb is functioning optimally. While the other factors are
relevant, the
previous ability of the patient to ambulate is the greatest concern

21. A patient has difficulty palpating the carotid pulse during exercise. The patient
should be instructed in alternate methods of self-monitoring, because repeated
palpation is likely to result in
1/1

A. increasing the heart rate.


B. decreasing the heart rate.
C. an irregular heart rhythm.
D. increasing systolic blood pressure.
Feedback
B. Pressure receptors (baroreceptors) are present in the carotid sinus and these receptors respond
to changes in
blood pressure. An increase in blood pressure that is sensed by these receptors will stimulate the
parasympathetic
system to decrease the rate and force of contraction of the heart in order to help lower the pressure.
Repeated
palpation in the carotid sinus area may simulate an increase in blood pressure and cause this
reaction. Therefore
increased heart rate and blood pressure are incorrect. Irregular heart rhythms generally result from
electrolyte
imbalance and/or ischemia to the conduction system of the heart.

22. A physical therapist plans to study the effect of cold compresses on passive
range of motion in a group of 10 patients. The plan is to apply these compresses to the
hamstring muscles 1 time/day for 5 days. Which of the following experimental designs is
MOST appropriate for this type of study?
0/1

A. For both the experimental and control groups, gather data from patient records.
B. For both the experimental and control groups, measure range of motion of both groups on
day 5.
C. For both the experimental and control groups, measure range of motion on days 1 and 5.
D. For the experimental group, measure range of motion every day. For the control group,
measure range of motion on days 1 and 5

23. While a patient is walking in the parallel bars, the physical therapist observes that
the pelvis drops down on the side opposite the stance extremity. This gait deviation is
an indication of weakness of the hip:
1/1

A. abductors of the swing extremity


B. adductors of the swing extremity
C. abductors of the stance extremity.
D. adductors of the stance extremity.

24. When treating a patient who has ankylosing spondylitis, the muscles requiring
the MOST emphasis for strengthening exercises are the:
1/1

A. pectorals.
B. hip flexors.
C. back extensors
D. abdominals.

25. A patient with Parkinson’s disease has just been admitted to the rehabilitation
unit. The patient is dependent in all transfers and requires moderate assistance of 1
person to walk 30 ft (9.1 m) with a standard walker. To facilitate good carry-over for
activities, instruction of the family in transfers should occur:
1/1

A. during a home visit after the patient is discharged


B. just prior to discharging the patient
C. early in the rehabilitation program
D. when the family feels ready to take the patient home

26. A patient has a history of neck pain that is aggravated by long periods of sitting.
The pain becomes progressively worse by evening. Range of motion and strength of the
neck and shoulders are within normal limits. Sensation and reflexes are intact in both
upper extremities. The patient has a forward head and excessive thoracic kyphosis. The
MOST appropriate exercise program includes:
1/1

A. stretching of the neck flexors and pectoral strengthening


B. upper trapezius strengthening and pectoral stretching.
C. pectoral strengthening and rhomboid stretching
D. rhomboid strengthening and axial neck extension

27. To achieve maximum reduction of lymphedema following a mastectomy in the


upper extremity by means of massage, it is MOST important that:
···/1

A. local heat be applied before the massage


B. the upper arm be massaged before the forearm
C. the hand be massaged before the forearm.
D. the massage strokes occur in a centrifugal direction

28. Which of the following objectives is MOST important prior to discharge, for a
patient who has had a myocardial infarction?
0/1
A. Ascend a flight of stairs before discharge
B. B. Perform prescribed exercises without angina
C. Return to normal daily activity level.
D. Take a radial pulse reliably before discharge

29. A physical therapist who works in a home health agency is treating a patient with
diabetes mellitus. The patient tells the therapist that he is no longer taking his insulin.
The physical therapist’s FIRST course of action should be to:
0/1

A. instruct the patient in the proper technique for injection of insulin


B. contact the patient’s home health nurse.
C. tell the patient’s family to report this information to the physician.
D. have the patient perform a urine glucose test while the therapist is in the home.

30. Which of the following instructions is MOST appropriate for teaching a patient
with C6 quadriplegia to transfer from a wheelchair to a mat?
1/1

A. Keep your fingers extended to give a broader base of support


B. Rotate your head and shoulders in the same direction as the desired hip motion.
C. Rotate your head and shoulders in the direction opposite to the desired hip motion.
D. Keep both hands next to your knees to lock your elbows.

31. A patient, who recently had an ankle injury, reports pain in the front of the ankle
while walking. This pain is reproduced when, with the patient in sitting, the physical
therapist grasps the patient’s heel and with the foot remaining in neutral introduces a
lateral (external) rotation force to the lower extremity. The pain is not reproduced with
inversion or eversion tests to the ankle. The patient MOST likely has which of the
following injuries?
1/1

A. Syndesmosis sprain
B. Calcaneocuboid sprain
C. 5th metatarsal fracture
D. Tibial stress fracture
32 A patient with low back and leg pain has been seen 2 times in outpatient physical
therapy. Today, during the patient’s third visit, the therapist notes an audible slap of the
foot at heel strike (initial contact). The patient also reports that there has been no leg
pain since yesterday. Regarding testing, it is MOST important for the physical therapist
to perform
1/1

A. Achilles’ tendon reflex testing.


B. sensation testing of the sole of the foot.
C. the sign of the buttock test.
D. tibialis anterior strength testing

33. A patient, who has fallen 3 times while walking on uneven surfaces, comes to
physical therapy for intervention. Examination reveals that the patient does not have
difficulty maintaining balance with the eyes closed or with the introduction of visual
conflict. Which of the following interventions is MOST appropriate for this patient?
1/1
A. Instruct the patient to walk a straight line while glancing left and right.
B. Instruct the patient in exercises to strengthen the gastroc-soleus muscle group.
C. Refer the patient to an orthotist for a custom ankle-foot orthoses fitting
D. Provide the patient with a cane for walking

34. A patient has had left Achilles’ tendinitis for 6 weeks. No structural abnormalities
are evident upon examination of the patient. Which of the following supportive shoe
inserts is MOST appropriate for this patient
0/1

A. Full-contact custom orthosis, left only


B. Off-the-shelf heel lift, left only
C. Off-the-shelf heel lifts, bilaterally
D. Full-contact custom orthoses, bilaterally

35. A physical therapist is working with an outpatient who had a cerebrovascular


accident and currently lives in an assisted-living facility. Which of the following
statements is the MOST appropriate functional goal for this patient?
1/1

A. The patient will be able to don an ankle-foot orthosis with assistance


B. The patient will independently walk 165 ft (50 m) with a straight cane from the bedroom to
the cafeteria.

C. The patient will have Normal (5/5) strength of the quadriceps muscles
D. The patient’s balance will improve to be able to independently stand on the involved lower
extremity for 20 seconds

36. A physical therapist is working on transfers with a patient who had a brainstem
cerebrovascular accident. The patient has ataxia in all 4 extremities and a high level of
extensor tone in the lower extremities. The patient has fair to good trunk control. Which
of the following transfers is BEST for this patient?
1/1

A. Squat-pivot

B. Sliding-board
C. Standing-pivot
D. Dependent tuck

37. A patient with an L4 - L5 posterolateral herniated nucleus pulposus is MOST


likely to have sensory deficits in which of the following locations?
0/1
A. Medial knee
B. Medial ankle
C. Plantar aspect of the foot
D. Dorsum of the great toe

38. A patient had a brainstem stroke 2 months ago. The patient is currently able to
independently walk 65 ft (20 m) over level surfaces with a straight cane and ascend
stairs with minimum assistance. Which of the following activities would MOST
appropriately challenge this patient’s balance during a physical therapy session?
0/1

A. Ascending stairs using a single handrail

B. Standing on 1 leg with eyes closed


C. Walking over uneven terrain
D. Walking 130 ft (40 m) with a straight cane

39. A patient who recently had bilateral leg amputations wants to have a ramp built to
travel from the back deck of his house to the pool in his wheelchair. The vertical
distance from the door to the ground level is 5 ft (1.5 m). Which of the following ramp
specifications is BEST for this patient?
1/1

A. 1 continuous ramp, 60 ft (18 m) long


B. 1 continuous ramp, 30 ft (9 m) long
C. 2 ramps, each 60 ft (18 m) long, connected by a level area
D. 2 ramps, each 30 ft (9 m) long, connected by a level area

40. The results of pulmonary function tests of a patient with which of the following
diseases is LEAST likely to show increased residual volume?
0/1
A. Atelectasis
B. Bronchiectasis
C. Chronic bronchitis

D. Emphysema

41. A patient reports numbness and tingling on the lateral side of the right lower leg
that sometimes extends into the top of the foot. Which of the following structures is
MOST likely involved?
0/1

A. Lateral sural cutaneous nerve


B. L4 nerve root
C. Superficial peroneal nerve
D. S1 nerve root

42. Which of the following outcomes is the HIGHEST expected functional outcome
for a patient with a complete C7 spinal cord injury?
1/1

A. Minimal assistance with transferring from the floor to the wheelchair


B. Independent with wheelchair mobility on smooth surfaces
C. Minimal assistance with transferring from supine to sitting
D. D. Independent with ascending a curb in the wheelchair

43. In splinting or immobilization, the functional position of the hand includes wrist:
1/1
A. extension, phalangeal flexion, and abduction of the thumb (1st digit)

B. extension, phalangeal flexion, and abduction of the thumb (1st digit)


C. extension, phalangeal flexion, and adduction of the thumb (1st digit).
D. extension, phalangeal flexion, and adduction of the thumb (1st digit

44. Which of the following assignments is MOST appropriate for a physical therapist
to delegate to a volunteer?
1/1

A. Restocking treatment booths with linens, ultrasound gel, and massage lotion

B. Attending a patient who is on a tilt table while the therapist takes a phone call
C. Transporting a patient who reports dizziness back to the patient's room
D. D. Transferring a patient from the mat table to a wheelchair

45. A client on a weight-loss program has been walking 3 days/week for 15 minutes, for
the past 3 weeks. To progress the exercise program, which of the following will MOST
likely accomplish the weight-loss goal?
1/1

A. Maintain the current walking speed and increase the duration to 30 minutes

B. Increase the walking speed and keep the duration at 15 minutes.


C. Walk 4 days/week and decrease the duration to 10 minutes.
D. Change from walking 3 days/week to jogging 1 day/week for 20 minutes.

46. During the shoulder examination of a patient, a physical therapist notes the
presence of a capsular pattern without radicular pain. To help establish the cause of the
capsular pattern, the therapist should NEXT
0/1
A. perform axial compression on the cervical spine to check for nerve root compression
B. ask the patient if there has been any prior trauma to the shoulder joint.
C. check for a painful arc during active range of motion
D. examine the shoulder for a rotator cuff tear

47. A physical therapist is assessing the lifting technique of a patient who has a
history of back pain. The patient performs a lift by picking up a light weight from the
floor, without bending the knees. During the lift, the therapist notes that the patient
demonstrates excessive lumbar flexion. Limitation of which of the following measures is
the STRONGEST contributor to this finding?
1/1

A. Hamstring flexibility

B. Gluteal muscle strength


C. Abdominal muscle strength
D. Hip flexor flexibility

48. A physical therapist is evaluating a patient who had a stroke and is exhibiting
poor foot and ankle control. When asked to lift the foot more during the midswing phase
of gait, the patient dorsiflexes the ankle with excessive eversion. Facilitatory
electromyographic biofeedback to which of the following muscles is MOST likely to help
correct this problem?
1/1
A. Extensor digitorum longus
B. Tibialis anterior

C. Fibularis (peroneus) brevis


D. Flexor hallucis longus

49. A patient presents with weakness throughout the right lower extremity with
normal strength on the left side. Sensory testing shows a loss of pressure sensation
over the right thigh and leg, and a loss of pain and temperature sensation over the left
thigh and leg. The patient has a positive Babinski’s sign present on the right. Which of
the following associated findings is MOST likely to be found during further examination
of this patient?
1/1

A. The presence of clonus in the left ankle


B. Marked atrophy in the right lower extremity muscles
C. Spasticity in the left lower extremity
D. Increased deep tendon reflexes on the right side

50. A patient was injured in a hunting accident, sustaining a gunshot wound to the
spine in the area of L1. The patient has weakness of the left lower extremity and
inability to move the knee, ankle, or foot. The patient’s patellar tendon and Achilles’
tendon reflexes are increased on the left side. There is also loss of proprioception in the
patient’s left ankle and knee, a positive Babinski’s sign on the left side, and diminished
sensation to pinprick and temperature changes in the right thigh, leg, and foot. Results
of all of the patient’s cranial nerves’ tests are normal. These findings are consistent with
which of the following injuries?
1/1

A. Complete severance of the spinal cord


B. Injury to the left anterior horn of the spinal cord
C. Injury to the left side of the spinal cord

D. Injury to the central area of the spinal cord

1. A herniated nucleus pulposus at the L3-L4 disc produces which of the following
clinical findings?
1/1
A. Numbness in the back of the calf and dorsiflexor weakness
B. Numbness in the anteromedial thigh and knee and quadriceps weakness

C. Numbness and weakness in the thigh, legs, feet and/or perineum


D. Numbness in the back of the calf and atrophy of gastrocnemius and soleu
Feedback
B. Pressure on the L3 L4 root causes numbness in the anteromedial thigh and knee and quads
weakness.
Numbness in the back of the calf and dorsiflexor weakness is indicative of L4, L5 and S1 nerve root
involvement.
Numbness and weakness in the thighs, legs and feet and perineum is indicative of L2 through S1
nerve root
involvement. Numbness in the back of the calf and atrophy of gastroc soleus is indicative of the S1
nerve root
involvement

2. Which of the following techniques is MOST appropriate for a patient with low postural
tone?
0/1

A. Slow regular rocking while sitting on the treatment bolster


B. Continuous pressure to the skin overlying the back muscle

C. Low frequency vibration to the back muscle


D. Joint approximation applied through the shoulder to the trunk

Correct answer
D. Joint approximation applied through the shoulder to the trunk

3. A patient with complete long thoracic nerve injury would have difficulty in:
1/1

A. putting hand in the back pocket.


B. bending toward the involved side.
C. taking a deep breath.
D. reaching forward above head

Feedback
D. The long thoracic nerve innervates the serratus anterior muscle. Reaching forward above the head
(shoulder
flexion) would require the action of serratus anterior, and long thoracic nerve injury would affect the
motion. The
serratus anterior muscle also works in conjunction with the upper and lower trapezius muscle to
upwardly rotate
the scapula (abduct) during shoulder flexion. Putting the hand in the back pocket would require
scapular
adduction. Side bending and deep breathing would not require scapular motion.

4. A patient who recently and successfully completed a 12 week program of phase III
cardiac rehabilitation will MOST likely demonstrate a decrease in:
0/1

A. CO2 elimination in maximal work


B. cardiac output in maximal work

C. stroke volume at a given level of submaximal work


D. heart rate at a given level of submaximal work

Correct answer
D. heart rate at a given level of submaximal work
Feedback
D. Aerobic conditioning that occurs during the 12 weeks of cardiac rehab will result in a decrease HR
both at rest
and with exercise. CO2 elimination and cardiac output would both increase with maximal work. The
stroke
volume would increase during submaximal work

5. Rocking in quadruped position could be included in physical therapy intervention for a


40 year old patient to increase ROM at the:
1/1
A. hip joint, 1 month post total hip replacement
B. distal radioulnar joint, 2 weeks post fracture of the distal radius
C. glenohumeral joint, 2 months post rotator cuff tear

D. hip joint, 1 week post subcapital fracture of the femoral nec


Feedback
C. Increasing hip flexion is usually contraindicated for THA at 1 month. The quadruped position
would not be
allowed for a patient at 2 weeks post distal radius fracture. There are no precautions for the patient
2 months
following a rotator cuff tear as the motions in quadruped rocking would be synonymous with the
glenohumeral
motions that need to be increased and are achieved in a closed chain position.

6. A therapist is treating a young athlete with gastrocnemius muscle strength of fair plus
(3+/5). In the prone position, which of the following exercises is the MOST appropriate
to maximize strengthening?
0/1

A. Resistive exercises with the knee bent.

B. Resistive exercises with the knee straight.


C. Active exercises with the knee bent
D. Active exercises with the knee straight.

Correct answer
B. Resistive exercises with the knee straight.

7. A therapist is evaluating a patient with an acute lumbar disc protrusion and a right
lateral shift of the thoracic spine. Which of the following findings would be the BEST
indicator that the symptoms will respond positively to the PT intervention?
1/1
A. The pain is referred only to the buttock and not the thigh.
B. The patient prefers standing and walking than sitting.
C. There is a decrease in lumbar lordosis.
D. Repeated backward bending centralizes the pain.

Feedback
D. The fact that backward bending is centralizing the pain is an indicator that the nucleus of the disc
is being
moved centrally and the pressure is being taken off the nerve root. Centralization of the pain is also
a good
indicator for PT intervention.

8. A 90-year-old patient with chronic congestive heart failure has been non-ambulatory
and a nursing home resident in the past year. The patient was recently admitted to the
hospital following an episode of dehydration. Which of the following plans for
prophylactic respiratory care is MOST appropriate?
1/1

A. Turning, coughing and deep breathing every 1 to 2 waking hours.

B. Vigorous percussion and vibration 4 times a day


C. Gentle vibration, with the foot of the bed elevated once a day.
D. Segmental postural drainage using standard positions throughout the day
Feedback
A. A patient who is bed bound and immobile will be prone to developing atelectasis (partial collapse
of lung
tissue); which can lead to pneumonia. Frequent position changes with deep breathing and coughing
will help
prevent development of atelectasis. Given that this patient is elderly and does not have diagnosis of
secretion
retention, vigorous percussion and vibration is not indicated. Vibration with the head down or
standard postural
drainage positions will not be tolerated in this elderly patient with chronic CHF.

9. Which diagnostic procedure is LEAST likely to confirm suspected disc herniation in a


patient with low back pain?
0/1

A. MRI scan
B. CAT scan
C. Radiograph
D. Myelogram
Correct answer
C. Radiograph

10. To help the students apply newly learned skill to clinical practice, the MOST
effective action for the clinical instructor to take is to:
0/1

A. point our possible patient situations and discuss how the skill would apply to them.
B. have the students research reference materials and compile a list of the steps required in the
acquisition of the

skill
C. prepare a list of indications and contraindications for the skill.
D. have the students provide examples of patient situations where the skill would be
appropriately applied.

Correct answer
D. have the students provide examples of patient situations where the skill would be
appropriately applied.

11. A patient is referred to physical therapy complaining of severe pain in the right hip
and groin area, which increases during walking. The patient complains of tenderness
when the therapist palpates the area over the right greater trochanter. The MOST likely
cause of the patient’s signs and symptoms is:
1/1

A. sacroiliac joint derangement


B. a hip fracture
C. a strain of the adductor longus muscle
D. hip bursitis

Feedback
D. Signs and symptoms of hip bursitis include the following: severe pain over the bursa area, with
pain
aggravated by active motion including activities such as walking. Signs and symptoms of a
sacroiliac joint
derangement include pain directly over the region of the joint and in the low back, in addition to pain
with
walking. Signs and symptoms of hip fracture will include the following: severe pain in the groin area
and
tenderness occurs in the area anterior to the femoral neck. An adductor longus muscle strain would
not cause
tenderness over the greater trochanter
12. Which of the following instructions is MOST appropriate for teaching a patient with
C6 quadriplegia to transfer from a wheelchair to a mat?
0/1

A. “Keep your fingers extended, to give a broader base of support”

B. “Rotate your head and shoulders in the same direction to the desired hip motion”
C. “Rotate your head and shoulders in the direction opposite to the desired motion”
D. “Keep both hands next to your knees to lock your elbows”

Correct answer
C. “Rotate your head and shoulders in the direction opposite to the desired motion”

13. The MOST appropriate therapeutic exercise to stretch the neck muscles for the
patient with an acute right sided torticollis is:
1/1

A. right rotation and right lateral flexion


B. left rotation and right lateral flexion
C. left rotation and left lateral flexion
D. right rotation and left lateral flexion

Feedback
D. Isolated contraction of the right sternocleidomastoid muscle would cause the head to rotate the
left and side
bend (lateral flexion) to the right. To stretch the muscle, the opposite actions would be performed on
the patient
by the therapist; rotation of the head to the right and lateral flexion to the left.

14. A therapist who works in a home health agency is treating a patient with diabetes
mellitus. The patient tells the therapist that he is no longer taking his insulin. The
therapist’s FIRST course of action should be to:
0/1

A. instruct the patient in proper technique for injecting insulin


B. contact the patient’s home health nurse
C. tell the patient’s family to report this information to the physician

D. have the patient perform a urine glucose test while the therapist is in the home

Correct answer
B. contact the patient’s home health nurse
15. Which of the following is the MOST appropriate intervention for a patient with
juvenile rheumatoid arthritis who is experiencing painful swelling of both knees?
1/1

A. Resistive exercises
B. Stretching to prevent contractures
C. Gentle active exercises

D. Walking program
Feedback
C. All of the options, except for gentle, active exercises, are precautions or contraindications for this
patient

16. A patient who is on bed rest exhibits orthostatic hypotension upon standing will
MOST likely experience:
1/1

A. increase venous tone in the lower extremity


B. inadequate ventricular filling during diastole

C. decrease hydrostatic pressure in the capillary beds


D. parasympathetic stimulation of the heart
Feedback
B. Orthostatic hypotension from bed rest occurs as a result of decreased venous tone, which will
lead to a pooling
of blood in the LE upon standing. The pooling of blood in the LE will reduce the amount of blood
returning to the
heart decreasing ventricular filling and ultimately decreasing cardiac output. This results in drop in
BP with
resultant dizziness. With pooling of blood in the LE, the hydrostatic pressure would increase. To
compensate for
the decrease in cardiac output, the sympathetic system (not the parasympathetic) would stimulate
the heart

17. A patient is undergoing a treadmill stress test. The appearance of abnormally wide,
irregularly spaced QRS complexes on the ECG represents:
0/1

A. ventricular depolarization

B. premature ventricular contractions


C. atrial fibrillation
D. atrial repolarization

Correct answer
B. premature ventricular contractions

18. Which of the following techniques is MOST important prior to discharge for a patient
who has had myocardial infarction?
0/1

A. Ascend a flight of stairs before discharge


B. Perform prescribed exercises without angina
C. Return to normal daily activity level

D. Take a radial pulse reliably before discharg

Correct answer
D. Take a radial pulse reliably before discharg

19. A physical therapist is designing an exercise program for a healthy, elderly


individual. Which of the following types of exercise is MOST stressful to the
cardiovascular system?
1/1

A. Upper limb ergometry

B. Spinal ROM exercise


C. Quadriceps setting exercise
D. Lower limb diagonal PNF pattern
Feedback
A. Heart rate and systolic blood pressure responses are higher for any given workload when
performed with the
UE as compared to the LE. Spinal motion does not create much stress on the cardiovascular system.
The
quadriceps setting and diagonal patterns are performed with the LE and would produce less
cardiovascular stress

20. A patient with a diagnosis of complete SCI at C4-C5 demonstrates a weak cough
mechanism. Which of the following techniques would MOST effectively augment
coughing?
0/1
A. Manual pushing against the upper abdomen
B. Positioning the patient in prone

C. Pursed-lip breathing
D. Interrupting the expiratory air stream

Correct answer
A. Manual pushing against the upper abdomen

21. A physical therapist is teaching a patient with T12 paraplegia to fall. The patient
walks with bilateral KAFO and bilateral forearm crutches using swing to gait. The MOST
appropriate method to use to prevent injury would be to:
1/1

A. have the patient wear a helmet and practice on a soft padded surface
B. give the patient written instructions and illustrations on falling
C. demonstrate and guide the patient through the correct process of falling

D. demonstrate the correct method of falling


Feedback
C. If patients are skilled in falling, they are less likely to become injured during a fall. The patient
demonstrating
or performing the task best understanding of a skill. To accomplish this, the patient should be given
a
demonstration of proper falling technique and then the patient should be allowed to practice while
being guided
and guarded by the therapist. Without first demonstration and then allowing the patient to practice,
the patient
could be place at risk for injury. Therefore, options A, B & D are not adequate to ensure patient
safety.

22. A 6-year-old child with spastic diplegia is walking in the parallel bars. The child
walks with increased trunk and hip flexion. What would be the MOST appropriate
assistive device?
0/1

A. Standard walker

B. Forearm crutches
C. Posterior rolling walker
D. Bilateral quad canes

Correct answer
C. Posterior rolling walker
23. A patient with complete C4 quadriplegia is working on a program to increase
tolerance to the upright position. While on the tilt table, the patient begins to complain of
a pounding headache, with flushing and profuse sweating. The therapist should FIRST
0/1

A. lower the tilt table to a flat position

B. remove the patient from tilt table and return to room


C. check the patient’s catheter
D. check the patient’s BP

Correct answer
C. check the patient’s catheter

24. A therapist is preparing to teach a patient who has Guillain-Barre syndrome to


transfer from wheelchair to a mat table. The patient weighs 150 lbs., she has poor (2/5)
grade in both of her lower extremities and fair (4/5) grade in her left upper extremity; her
right upper extremity has normal (5/5) strength. Which of the following would be the
MOST appropriate assisted transfer for the initial session of this patient?
1/1

A. Two person lift on her right side


B. Sliding board transfer to her right side

C. A hydraulic lift from her wheelchair to the mat


D. Full, standing pivot transfer to her right side
Feedback
B. Because this patient has significant weakness in both lower extremities, a standing pivot transfer
would not be
safe. However, since the patient does have fair to normal upper extremity strength, and is not totally
dependent, a
hydraulic lift or two person lift would not be appropriate because it would not be enhancing the
patient’s
functional skills. The most appropriate transfer method would be for the patient to use a sliding
board transfer to
her strongest right side.

25. Upon removal of a wet-to-dry dressing from a draining wound, the therapist
observes that the skin immediately surrounding the wound is macerated. What should
the therapist recommend for future wound care?
1/1
A. Continue using current dressing type
B. Make the dressing more absorbent

C. Leave the dressing on longer between dressing changes


D. Change to a pressure type dressing
Feedback
B. Macerated tissue results from excessive moisture. A more absorbent dressing would soak up the
excessive
moisture and prevent maceration.

26. A patient has a history of neck pain that is aggravated by long periods of sitting. The
pain becomes progressively worse by evening. ROM and strength of the neck and
shoulder are within normal limits. Sensation and reflexes are intact in both UE. The
patient has a forward head and excessive thoracic kyphosis. The MOST appropriate
exercise program would include:
1/1

A. stretching of the neck flexors and pectoral strengthening


B. upper traps strengthening and pectoral stretching
C. pectoral strengthening and rhomboid stretching
D. rhomboid strengthening and axial neck extension

Feedback
D. The history suggests that prolonged improper positioning of the cervical spine resulted in neck
pain. A chronic
forward head and kyphosis results in hyperextension in the upper cervical spine and excessive
flexion in the upper
thoracic spine. Further muscle length adaptation occurs with tight anterior muscles and stretched
posterior
muscles. Treatment should include correction of muscle weakness or imbalance. Rhomboid
strengthening and
axial neck extension are the only options that are both correct

27. A patient with left hemiparesis is being discharged from the rehabilitation center.
The therapist has been unsuccessful in getting the family’s cooperation to instruct them
in transfers, bed mobility, and safety precautions. The MOST appropriate action for the
therapist to take is to:
1/1

A. educate the patient so she can tell others how to assist her
B. contact social services and arrange for a team and family meeting
C. refer the patient for home care and document appropriate interventions
D. write a home program and give it to the family
Feedback
B. Of the options above, a social worker (social services) would be the most appropriate person to
intervene in
this case. The social worker is trained to help resolve family issues that arise and would be the best
person to lead
a team of caregivers and family members in deciding the appropriate action in this case. Educating
the patient
would only put more burden on them without guaranteeing results. A referral for home care may be
appropriate,
but the social worker would be the best person to make such contact. Writing a home program for
the family
would not address the need to instruct family members in person

28. Which of the following should be recommended for a patient following a medial
meniscectomy if the PT plans to exercise the quadriceps femoris against
accommodating resistance?
0/1

A. Knee bends through half of the range


B. Knee extension with sandbag in the ankle
C. Knee extension on an isokinetic exercise device
D. Straight leg raising with a sling suspensionn 1

Correct answer
C. Knee extension on an isokinetic exercise device

29. A patient with chronic venous insufficiency of the LE would MOST likely exhibit:
1/1
A. normal superficial veins, no edema, ulceration and patches of gangrene around the toes
B. dilation of superficial veins, edema, stasis ulceration

C. no edema, cold, hairless LE, faint dorsalis pedis pulse


D. dilation of superficial veins, edema made worse during sitting or leg elevatio
Feedback
B. With venous insufficiency, the limbs would be edematous, the superficial veins would be dilated
and if not
corrected ulceration could develop. Options A and C are ruled out because they indicate no edema.
Option D is
not correct because the condition is relieved by sitting or leg elevation.
30. Which lower extremity PNF pattern is MOST appropriate for a patient who needs
strengthening of the tibialis posterior?
1/1

A. Hip extension, abduction and medial rotation with ankle plantarflexion and eversion
B. Hip flexion, adduction and lateral rotation with ankle dorsiflexion and inversion
C. Hip extension, adduction and lateral rotation with ankle plantarflexion and inversion

D. Hip flexion, abduction and medial rotation with ankle dorsiflexion and eversion
Feedback
C. The tibialis posterior plantarflexes and inverts the foot. This pattern requires the specific action of
that muscle.
The other patterns do not

30. An adult patient who was involved in a motor vehicle accident has sustained
multiple traumas, including fractured ribs on the right side. The patient is unconscious,
intubated and on a mechanical ventilator in the ICU. Chest radiographs show the
development of an infiltrate in the lower lobe during the past two days. Rales and ronchi
are heard over the right lower lung fields. Which chest program is MOST appropriate?
0/1

A. Manual hyperventilation and suctioning while positioned on the left side


B. Positioning supine for suctioning, followed by manual hyperventilation while positioned on
the left side

C. Suctioning, percussion and vibration while positioned on the right side


D. Positioning on the left side for deep breathing exercises only

Correct answer
A. Manual hyperventilation and suctioning while positioned on the left side

31. A PT is examining a patient who has difficulty making a fist. The muscle test show
the following: flexors of the 2nd to 5th digits at the PIP are normal (5/5); flexors of the
2nd and 3rd digits at the DIP are normal (5/5); flexors of the 4th and 5th digits at the DIP
are poor (2/5). These findings are consistent with a lesion of the
1/1

A. median nerve C8-T1


B. median nerve C5-C6
C. ulnar nerve C5-C6
D. ulnar nerve C8-T1
Feedback
D. The weakness in the flexors of the 4th and 5th digits at the DIP joint (2/5). The other fingers show
normal
strength (5/5). This weakness would indicate a problem with the flexor digitorum profundus, since
the distal joints
are involved. The FDP is innervated by both median and ulnar nerves. The median nerve innervates
the FDP
muscles flexing the index and middle fingers. The ulnar nerve innervates the FDP muscles flexing
the ring and
little fingers. Therefore, since the ring and little fingers are involved, the lesion would have to be
involving the
ulnar nerve. The FDP to the 4th and 5th fingers is innervated by the ulnar nerve roots C8 and T1

32. During LE exercise, a patient with incomplete C6 quadriparesis reports a new onset
of tingling and numbness in both hands. MMT reveals that overall strength has
decreased by one grade. In this situation, the therapist’s MOST appropriate course of
action is:
1/1

A. modify the exercise program to accommodate change in strength


B. apply an ice massage prior to exercise
C. continue LE therapeutic exercise and use facilitation techniques to stimulate more activity
D. discontinue all exercises and notify the physician

Feedback
D. Sudden tingling of the hands and weakness of muscle groups are alarming signs of cord
compression. The
concern is that the spinal cord sustaining further injury. The exercise may be exacerbating or
causing these signs.
This is considered a medical emergency and the physician should be immediately advised.

33. A PT is developing an educational program for individuals with LE peripheral


neuropathies due to diabetes. Which of the following is the MOST important information
for the PT to recommend for the prevention of injury to the foot?
1/1

A. Orthosis to support the extremity


B. Use of proper footwear

C. Moisturizing the skin to prevent dryness


D. Exercise parameter
Feedback
B. While all of the above would help prevent injury to a diabetic foot, the most important information
to provide
is about proper footwear. Ensuring properly fitting footwear would alleviate risk of skin breakdown
as well as
providing appropriate cushioning to the articular cartilage of the foot joints that are prone to injury
from repetitive
trauma and compression. Orthoses may not be necessary for all patients. Despite moisturizing the
skin, if there is
improper protection, skin breakdown can still occur. Exercising within appropriate parameters will
not be
beneficial if the patient’s footwear is not supportive or properly fitting.

34. While ascending stairs, an elderly patient leans forward with increased hip flexion.
Which of the following muscles are being used to the BEST advantage with this forward
posture?
0/1

A. Rectus femoris

B. Tensor fascia lata


C. Gluteus maximus
D. Lumbar paraspinal

Correct answer
C. Gluteus maximus

35. A PT recognizes that departmental education materials are written on a reading


level that is too difficult for some of the patients. The PT can help the patients better
understand the materials by:
1/1

A. including illustrations and a larger print


B. including illustrations and a simpler sentence structure

C. reducing the number of syllables and lengthening the sentences.


D. using a larger print format to present specific medical terminology
Feedback
B. A larger print will not simplify the meaning, nor will lengthening the sentences. Only option B, the
choice and
illustrations and a simpler sentence structure is correct
36. During inspiration, a patient demonstrates increased upper chest expansion with
retraction of the epigastric area. The PT should suspect weakness of the:
1/1

A. scalene muscle
B. diaphragm

C. rectus abdominis
D. intercostal muscles
Feedback
B. Normally, as the diaphragm descends with inspiration, it places increased pressure on the
abdominal contents
with a resultant distension of the epigastric area. Weakness of the diaphragm would decrease the
distension and
may result in a compensatory increase in upper chest expansion to improve ventilation. The scalene
muscles act to
elevate or stabilize the clavicle during inspiration. The rectus abdominis is a stabilizer during
inspiration and
assists with forced expiration. The intercostal muscles act to stabilize the thoracic cage during
ventilation.

37. The brother of a patient who was recently discharged from the hospital’s outpatient
PT department telephones on the patient’s behalf to request a copy of the patient’s
medical record. The therapist should explain to the patient’s brother that the medical
record is the property of the:
1/1

A. patient’s family and can be released to the bother upon request


B. patient’s insurer now and that the request for a copy must be made in writing to the insurer
C. hospital and the patient can be released only with written authorization from the patient

D. hospital and can be released only with written authorization from the patient’s physician
Feedback
C. The medical record is owned by the hospital subject to the patient’s interest in the information it
contains.
Unless restricted by state or federal law or regulation, a hospital shall furnish to a patient, or a
patient’s
representative parts of the hospital record upon request in writing by the patient or their
representative. Option C
is the only correct answer in this case, since it specifies that the patient must authorize the release
of information.
38. A PT is examining a patient for a possible LE weakness. PROM is within normal
limits. The patient is seated. When the patient tries to dorsiflex and invert his right foot,
he is unable to move it through the full ROM and is unable to take any resistance
applied by the therapist. During the subsequent gait examination, the therapist would
MOST likely expect the patient to display which of the following?
0/1

A. Normal gait, with no deviations


B. Increased right hip flexion during swing phase
C. Throwing the trunk backward, on the right side, shortly after the initial contact
D. Lateral bending toward the right side during midstance

Correct answer
B. Increased right hip flexion during swing phase

39. A patient with low back pain has been undergoing treatment for two sessions. The
patient tells the therapist that today the pain is centralizing with the extension exercises,
but is intense as it was at the first treatment session. The patient is frustrated by this
reaction. The therapist’s most appropriate response would be:
0/1

A. continue with the present program


B. eliminate the extension exercises

C. consult the patient’s physician about the situation


D. progress to trunk flexion exercises

Correct answer
A. continue with the present program

40. Which of the following is the MOST likely cause of reduced vital capacity in a patient
who has quadriplegia at C5-C6 level?
0/1

A. Decreased anterolateral chest expansion resulting from paralysis of the external intercostals
B. Inability of the patient to generate a negative intrapleural pressure secondary to a denervated
diaphragm
C. A relatively high resting position of the diaphragm resulting from paralysis of abdominal
muscles
D. Reduced rib cage elevation due to paralysis of the anterior scalene and sternocleidomastoid
muscles
Correct answer
A. Decreased anterolateral chest expansion resulting from paralysis of the external intercostals

41. Involving interdisciplinary team members in making decisions about continuity of


patient care has all of the following benefits EXCEPT:
0/1

A. promoting each individual’s commitment to, and responsibility for, the decisions that have
been made by the
group
B. discouraging one or two members from being in control and having all the authority

C. enabling decisions regarding quality of care to be made quickly


D. promoting quality of care by including several aspects of the patient’s care

Correct answer
C. enabling decisions regarding quality of care to be made quickly

42. In which of the following is independence the MOST realistic long-term goal for a
patient with a complete spinal cord lesion at C8?
1/1

A. Effective cough technique


B. Rolling from side to side
C. Transfer from bed to wheelchair

D. Ambulation with forearm crutches


Feedback
C. This patient would have functional use of the triceps and finger flexors and would therefore, be
independent
for all wheelchair transfers to all surfaces. Coughing would not be limited and would not be a goal.
Rolling
would be easily achievable as a short-term goal. Ambulation with forearm crutches would not be a
reasonable
expectation for this patient

43. When examining a patient who had a baby 3 days ago by vaginal delivery, the
physical therapist notices that the patient has a 2-cm. diastasis of the rectus abdominis.
To address this problem, the patient should:
1/1
A. perform partial sit-ups, supporting her abdominal muscles with her hands while lifting her
head in the supine

position
B. not perform exercises until the diastasis heals spontaneously
C. perform sit-ups with knees bent and arms behind the head
D. perform partial sit-ups with the legs straight and arms in front
Feedback
A. Raising just the head activates only the recti muscles, and supporting the abdominal muscle with
the hands
provides external support to the stretched abdominal muscles. This is the optimal position to initiate
strengthening
following delivery for a patient with diastasis rectus abdominis. Doing exercise would not be
appropriate. With
no support of the abdominal muscles, option C and D would be too aggressive at this time.

44. A patient is doing active and resistive exercises on a mat table in the physical
therapy department. After 15 minutes, the patient becomes short of breath, begins
coughing and expectorates pink, frothy sputum. At this point, the physical therapist
should first stop the treatment, then:
A. assess vital signs, let the patient rest a few minutes with the feet elevated, and ten resume
with a less vigorous
program
B. sit the patient up, assess vital signs and call a nurse or physician for further instructions

C. lay the patient supine, transfer the patient to stretcher and return the patient to the nursing
unit
D. lay the patient down flat, call for assistance and begin cardiopulmonary resuscitation
Feedback
B. The presence of Dyspnea and the pink, frothy sputum would suggest the presence of CHF and
resultant
pulmonary edema. CHF can occur from poor cardiac muscle function as a result of myocardial
infarction.
Pulmonary edema occurs from the backflow of blood from the heart into the pulmonary vessels,
increasing
pulmonary capillary pressure. The increase in pulmonary capillary pressure increases fluid
movement into the
alveoli, which are normally dry. These leads to the presence of pink, frothy sputum that can be
expectorated along
the shortness of breath (dyspnea). Positions that increase blood flow to the heart, such as lying flat,
will increase
the signs and symptoms. Therefore, the patient should be positioned with the head up or as in this
problem; the
patient should be placed in a sitting position to help alleviate the symptoms. Laying the patient down
flat, supine
or with the legs elevated would exacerbate the patient’s problems.

45. A patient who sustained a mild CVA 3 weeks ago is being prepared by the physical
therapist for discharge to home and an adult day program. To facilitate the discharge
plan, the MOST appropriate health professional for the therapist to consult with is the:
0/1

A. skilled nursing coordinator


B. occupational therapist
C. medical social worker
D. primary physician

Correct answer
C. medical social worker

46. To avoid the appearance of increased motion, which movement must be prevented
during goniometric measurement of shoulder abduction?
0/1

A. Upward rotation of the scapula


B. Medial rotation of the shoulder
C. Lateral rotation of the shoulder

D. Lateral flexion of the trunk to the opposite side

Correct answer
D. Lateral flexion of the trunk to the opposite side

47. A therapist examines a high school athlete in the training room. After removing the
adhesive strapping from the athlete’s ankle, the therapist discovers that the athlete has
developed an open weepy rash on the instep of the foot. The therapist should FIRST:
0/1

A. apply moist heat to the foot


B. send the patient to the hospital emergency room
C. use more pre-wrap with the next joint taping

D. refer the patient to the team physician

Correct answer
D. refer the patient to the team physician
48. A patient who sustained a left below-knee amputation 3 weeks ago is being
evaluated for possible ambulation with prosthesis. Which of the following factors is
MOST relevant?
0/1

A. Size of the right residual limb scar


B. Length of the right residual limb
C. Proficiency in previous prosthetic use
D. Severity of phantom pain

Correct answer
C. Proficiency in previous prosthetic use

49. A 14-month old child with brachial nerve palsy has motor and sensory loss in the
right upper extremity in the areas innervated by C5 & C6. Which of the following
activities is MOST difficult to accomplish with the affected upper extremity?
1/1

A. Pushing a wagon
B. Carrying a teddy bear in the crook of the arm

C. Holding a raisin in the palm of the hand


D. Grasping a cup
Feedback
B. The C5 and C6 myotomes include the biceps brachii, brachialis, brachioradialis and deltoid
muscles. The
elbow flexors and supinators (C5-C6) are used to carry a teddy bear in the crook of the arm. Pushing
a wagon
requires the use of the triceps (C7-C8). Holding a raisin requires the presence of wrist flexors (C6-
C8). Grasping
a cup requires finger, thumb and wrist muscles (C6-C8, T1).

50. A patient with TBI is receiving outpatient physical therapy. The therapist notices that
the patient becomes agitated during therapy sessions. To which of the following
professionals should the patient be referred for assessment and diagnosis?
1/1

A. Occupational therapist
B. Neuropsychologist

C. Vocation counselor
D. Speech-language pathologist
Feedback
B. Because the problem is a behavioral disorder, the most appropriate person to assess this patient
would be a
Neuropsychologist. The Neuropsychologist is trained to assess intellectual, emotional and coping
functions of the
patient as well as levels of depression and anxiety. They could also provide consultation to the
rehabilitation team
members as to how to deal with the problem

You might also like